В современном мире системы связи играют важную роль в развитие нашего мира. Каналы передачи информации буквально опутывают нашу планету, связывая различные информационные сети в единую глобальную сеть Интернет. Дивный мир современных технологий включает в себя передовые открытия науки и техники, не редко связанные также с удивительными возможностями квантового мира. Можно с уверенностью сказать, что на сегодняшний день квантовые технологии прочно вошли в нашу жизнь. Любая мобильная техника в наших карманах оснащена микросхемой памяти, работающая с использованием квантового туннелирования заряда. Подобное техническое решение позволило инженерами компании Toshiba построить 1984 году транзистор с плавающим затвором, ставшим основой для построения современных микросхем памяти. Мы каждый день пользуемся подобными устройствами, не задумываясь, на чем основана их работа. И пока физики ломают голову пытаясь объяснить парадоксы квантовой механики, технологическое развитие берет на вооружение удивительные возможности квантового мира.

В данной статье мы рассмотрим интерференцию света, и разберем способы построения канала связи для мгновенной передачи информации с применением квантовых технологий. Хотя многие полагают, что невозможно передавать информацию быстрее скорости света, при правильном подходе даже такая задача становится решаемой. Думаю, вы сами сможете в этом убедиться.

Введение


Наверняка многие знают о явлении под названием интерференция. Пучок света направляется на непрозрачную ширму с двумя параллельными прорезями, позади которого устанавливается проекционный экран. Особенность прорезей в том, что их ширина приблизительно равна длине волны излучаемого света. На проекционном экране получается целый ряд чередующихся интерференционных полос. Этот опыт, впервые проведенный Томасом Юнгом, демонстрирует интерференцию света, ставший экспериментальным доказательством волновой теории света в начале XIX века.

Логично предположить, что фотоны должны проходить сквозь щели, создавая две параллельные полосы света на заднем экране. Но вместо этого на экране образуется множество полос, в которых чередуются участки света и темноты. Дело в том, что когда свет ведет себя как волна, каждая прорезь является источником вторичных волн. В местах, где вторичные волны достигают экран в одной фазе, их амплитуды складываются, что создает максимум яркости. А там, где волны оказываются в противофазе — их амплитуды компенсируются, что создаст минимум яркости. Периодическое изменение яркости при наложении вторичных волн создает на экране интерференционные полосы.

Но почему же свет ведет себя как волна? В начале, ученые предположили, что возможно фотоны сталкиваются между собой и решили выпускать их поодиночке. В течение часа на экране вновь образовалась интерференционная картина. Попытки объяснить данное явление породило предположение, что фотон разделяется, проходит через обе щели, и сталкиваясь сам собой образует интерференционную картину на экране.

Любопытство ученых не давало покоя. Они хотели знать, через какую щель фотон проходит по настоящему, и решили понаблюдать. Для раскрытия этой тайны перед каждой щелью поставили детекторы, фиксирующей прохождение фотона. В ходе эксперимента выяснилось, что фотон проходит только через одну щель, либо через первую, либо через вторую. В результате на экране образовалась картина из двух полос, без единого намека на интерференцию. Наблюдение за фотонами разрушило волновую функцию света, и фотоны начали вести себя как частицы! Пока фотоны находятся в квантовой неопределенности, они распространяются как волны. Но когда за ними наблюдают, фотоны теряют волновую функцию и начинают вести себя как частицы.

Далее опыт повторили еще раз, с включенными детекторами, но без записи данных о траектории движения фотонов. Несмотря на то, что опыт полностью повторяет предыдущий, за исключением возможности получения информации, через некоторое время на экране вновь образовалась интерференционная картина из светлых и темных полос.

Получается, что влияние оказывает не любое наблюдение, а только такое, при котором можно получить информацию о траектории движения фотонов. И это подтверждает следующий эксперимент, когда траекторию движения фотонов отслеживается не с помощью детекторов установленных перед каждой щелью, а с помощью дополнительных ловушек, по которым можно восстановить траекторию движения не оказывая взаимодействия исходным фотонам.


Квантовый ластик


Начнем с самой простой схемы (это именно схематичное изображение эксперимента, а не реальная схема установки).


Отправим лазерный луч на полупрозрачное зеркало (ПП). Обычно такое зеркало отражает половину падающего на него света, а другая половина проходит насквозь. Но фотоны, будучи в состоянии квантовой неопределенности, попадая на такое зеркало, выбирают оба направления одновременно. Затем, каждый луч отражаясь зеркалами (1) и (2) попадает на экран, где наблюдаем интерференционные полосы. Все просто и ясно: фотоны ведут себя как волны.


Теперь попытаемся понять, по какому же именно пути прошли фотоны – по верхнему или по нижнему. Для этого на каждом пути поставим даун–конверторы (ДК). Даун–конвертор – это прибор, который при попадании в него одного фотона рождает 2 фотона на выходе (каждая с половиной энергии), один из которых попадает на экран (сигнальный фотон), а второй попадает в детектор (3) или (4) (холостой фотон). Получив данные с детекторов мы будем знать, по какому пути прошел каждый фотон. В этом случае интерференционная картина исчезнет, ведь мы узнали, где именно прошли фотоны, а значит, разрушили квантовую неопределенность.


Далее мы немного усложним эксперимент. Поставим на пути каждого «холостого» фотона отражающие зеркала и направим их на полупрозрачное зеркало (слева от источника на схеме). Так как «холостые» фотоны с вероятностью 50% проходят через такое зеркало или же отражаются от него, они с равной вероятностью попадут либо на детектор (5), либо на детектор (6). Не зависимо от того, какой из детекторов сработает, мы не сможем узнать по какому пути прошли фотоны. Этой замысловатой схемой мы стираем информацию о выборе пути и восстанавливаем квантовую неопределенность. В результате на экране будет отображаться интерференционная картина.

Если решим выдвинуть зеркала, то «холостые» фотоны вновь попадут на детекторы (3) и (4), и как мы знаем, на экране интерференционная картина исчезнет. Это означает, что меняя положение зеркал, мы можем менять отображаемую картину на экране. Значит, можно воспользоваться этим для кодирования двоичной информации.


Можно немного упростить эксперимент и получить тот же результат, двигая полупрозрачное зеркало на пути «холостых» фотонов:


Как мы видим, «холостые» фотоны преодолевают больше расстояния, чем их партнеры, которые попадают на экран. Логично предположить, если изображение на экране формируется раньше, то полученная картина не должна соответствовать тому, определяем ли траекторию «сигнальных» фотонов либо стираем эту информацию. Но практические опыты показывают обратное – не зависимо от расстояния, изображение на экране всегда соответствует производимым манипуляциям с холостыми фотонам. Согласно информации из википедии:
Основной результат эксперимента заключается в том, что не имеет значения, был процесс стирания выполнен до или после того, как фотоны достигли экрана детектора.
Подобный опыт также описывается в книге Брайана Грина «Ткань космоса и пространство». Это кажется невероятным, меняющим причинно-следственные связи. Попробуем разобраться что к чему.

Немного теории


Если посмотрим специальную теорию относительности Эйнштейна по мере увеличения скорости происходит замедление времени, согласно формуле:

где r – длительность времени, v – относительная скорость движения объекта.

Скорость света является предельной величиной, поэтому для самих частиц света (фотонов) время замедляется до нуля. Правильнее сказать для фотонов не существует времени, для них существует только текущий момент, в котором они пребывают в любой точке своей траектории. Это может казаться странным, ведь мы привыкли полагать, что свет от далеких звезд достигает нас спустя миллионы лет. Но с ИСО частиц света, фотоны достигают наблюдателя в тот же момент времени, как только они излучаются далекими звездами.

Дело в том, что настоящее время для неподвижных объектов и движущихся объектов может не совпадать. Чтобы представить время, необходимо рассмотреть пространство-время в виде непрерывного блока растянутого во времени. Срезы, формирующие блок, являются моментами настоящего времени для наблюдателя. Каждый срез представляет пространство в один момент времени с его точки зрения. Этот момент включает в себя все точки пространства и все события во вселенной, которые представляются для наблюдателя как происходящее одновременно.


В зависимости от скорости движения, срез настоящего времени будет делить пространство-время под разными углами. По направлению движению, срез настоящего времени смещается в будущее. В противоположном направлении, срез настоящего времени смещается в прошлое.


Чем больше скорость движения, тем больше угол среза. При скорости света срез настоящего времени имеет максимальный угол смещения 45°, при котором время останавливается и фотоны пребывают в одном моменте времени в любой точке своей траектории.

Возникает резонный вопрос, каким образом фотоны могут одновременно находится в разных точках пространства? Попробуем разобраться, что же происходит с пространством на скорости света. Как известно, по мере увеличения скорости наблюдается эффект релятивистского сокращения длины, согласно формуле:

где l – это длина, а v – относительная скорость движения объекта.

Не трудно заметить, что на скорости света любая длина в пространстве будет сжато до нулевого размера. Значит, по направлению движения фотонов пространство сжимается в маленькую точку планковских размеров. Можно сказать для фотонов не существует пространства, так как вся их траектория в пространстве с ИСО фотонов находится в одной точке.

Итак, теперь мы знаем, что не зависимо от пройденного расстояния сигнальные и холостые фотоны одновременно достигают экрана и наблюдателя, так как с точки зрения фотонов не существует ни времени ни пространства. Учитывая квантовую сцепленность сигнальных и холостых фотонов, любое воздействие на один фотон будет моментально отражается на состоянии его партнера. Соответственно, картина на экране всегда должна соответствовать тому, определяем ли мы траекторию фотонов, либо стираем эту информацию. Это дает потенциальную возможность моментальной передачи информации. Стоит только учесть, что наблюдатель не движется со скоростью света, и поэтому картину на экране необходимо анализировать после того, как холостые фотоны достигнут детекторов.

Практическая реализация


Оставим теорию теоретикам и вернемся к практической части нашего эксперимента. Чтобы получить картину на экране потребуется включить источник света и направить поток фотонов на экран. Кодирование информации будет происходить на удаленном объекте, движением полупрозрачного зеркала на пути холостых фотонов. Предполагается, что передающее устройство будет кодировать информацию с равными интервалами времени, например, передавать каждый бит данных за сотую долю секунды.


В качестве экрана можно использовать матрицу цифровой камеры, чтобы напрямую записывать на видео чередующиеся изменения. Затем записанную информацию необходимо отложить до момента, пока холостые фотоны достигнут своего местоназначения. Только после этого можно начать поочередно анализировать записанную информацию, чтобы получить передаваемую информацию. Для примера, если кодирующее устройство находится на Марсе, то анализ информации необходимо начинать с опозданием на десять-двадцать минут (ровно на столько, сколько требуется свету, чтобы достичь красную планету). Несмотря на то, что анализ информации производится с отставанием в десятки минут, полученная информация будет соответствовать тому, что передается с Марса в текущий момент. Соответственно, вместе с приемным устройством придется устанавливать лазерный дальномер, чтобы точно определить интервал времени, с которого нужно начинать анализировать передаваемую информацию.

Необходимо также учесть, что окружающая среда оказывает негативное влияние на передаваемую информацию. При столкновении фотонов с молекулами воздуха неминуемо происходит процесс декогеренции, увеличивая помеху в передаваемом сигнале. Чтобы максимально исключить влияние окружающей среды можно передавать сигналы в безвоздушном космическом пространстве, используя для этого спутники связи.

Организовав двухстороннюю связь, в перспективе можно построить каналы связи для моментальной передачи информации на любую дальность, до которых смогут добраться наши космические аппараты. Такие каналы связи будут просто необходимы, если потребуется оперативный доступ к сети интернет за пределами нашей планеты.

P.S. Остался один вопрос, которую мы постарались обойти стороной: а что случится, если мы посмотрим на экран до того, как холостые фотоны достигнут детекторов? Теоретически (с точки зрения специальной относительности Эйнштейна), мы должны увидеть события будущего. Более того, если отразить холостые фотоны от далеко расположенного зеркала и вернуть их назад, мы могли бы узнать собственное будущее. Но в реальности, наш мир куда более загадочнее, поэтому, трудно дать правильный ответ без проведения практических опытов. Возможно, мы увидим наиболее вероятный вариант будущего. Но как только мы получим эту информацию, будущее может измениться и возникнуть альтернативная ветка развития событий (согласно гипотезе многомировой интерпретации Эверетта). А возможно мы увидим смесь из интерференции и двух полос (если картина будет составлена из всех возможных вариантов будущего).

Полезные ссылки:
Walborn, S. P. (2002). «Double-Slit Quantum Eraser». Phys. Rev. A 65
Delayed choice quantum eraser. The experiment of Kim et al. (1999)
Эксперимент квантового ластика
Выступление Тома Кэмпбела
Квантовый ластик, предложенный Скалли и Дрюлем

Комментарии (272)


  1. flx0
    19.02.2018 17:24
    +2

    Пост в духе "найди ошибку в рассуждениях"? Что ж:


    Этой замысловатой схемой мы стираем информацию о выборе пути, а значит, восстанавливаем квантовую неопределенность.

    Ваш расщепитель луча переизлучает фотоны, то есть окончательно и бесповортно изменяет состояние системы. Волновая функция коллапсирует, запутанное состояние теряется. Обратно оно уже не восстановится.
    Впрочем, я не физик.


    1. Tyusha
      19.02.2018 20:18
      +1

      Нет, вы не правы, расщепитель ничего не переизлучает, тут претензий к автору нет. Когеренция теряется не здесь. Беда в том, что приходящие на расщепитель фотоны 3 и 4 уже не когерентны. Когеренция теряется ещё в даун-конверторах. Поэтому ни о какой интерференции ни на экране, ни в детекторах 5 и 6 речи быть уже не может.


      1. Tyusha
        19.02.2018 22:01
        +1

        Собственно после этого дальше схему обсуждать смысла нет, она нерабочая. Поясню: два разных прибора (даун-конветоры) создают разные фотоны. С какого перепугу они будут когерентны: как те, что падают на экран, так и те, что возвращаются назад. Вопрос закрыт.


        1. Gibboustooth
          20.02.2018 00:24

          Это же принципиальная схема. Ничего не мешает использовать 1 даун-конвертор.


          1. Smbdy_kiev
            20.02.2018 10:11

            Сама суть того, что первичный фотон был поглащён хотя бы одним конвертором (а потом да, рекомбинация, рассеивание....) изменяет когерентность. В том-то и вся фишка, что нельзя померить, не повлияв. А повлияв уже меняешь. Не стал дочитывать доконца, где информация передаётся быстрее скорости света (где? в вакууме?), простите меня грешного.


            1. Tyusha
              20.02.2018 12:20

              Зря. Я вот благодарна автору за интересный материал к размышению и попытку предложить оригинальную схему.


              1. NIN
                20.02.2018 13:06

                Мне кажется тут дело не в даун-конверторах. Принципиально ничего не мешает создать состояние .
                Дело в том, что если не трогать холостые фотоны, то интерференционной картины у сигнальных фотонов все равно не будет, так как их подсистема находится в смешанном состоянии. Просто так забыть о холостых фотонах и считать, что сигнальные фотоны будут интерферировать (т.е. чистое состояние) нельзя. Подробнее см. Нильсен и Чанг про редуцированный оператор плотности.


                1. zuborg
                  20.02.2018 15:04

                  Рассматриваемое в эксперименте состояние все-таки такое:
                  |Х0>|С0> + |Х1>|С1>
                  (нормирующий множитель опускаю) — либо Х и С фотоны оба идут по верхнему пути (0), либо оба по нижнему (1) (исходное состояние |S> после ПП-зеркала стало |S0> + |S1>, после ДК каждый из вариантов перешел из одного в пару фотонов)


                  1. NIN
                    20.02.2018 22:01

                    Согласен, можно, и гораздо проще так записать. Но это то же состояние, только записанное по-другому.


            1. Gibboustooth
              20.02.2018 13:46

              Я не уверен, но подозреваю, что, поглотив пары когерентных фотонов, даун-конвертер излучает пары также когерентных фотонов. Иначе непонятно, как из них потом «извлекают» интерференцию в оригинальном эксперименте.

              Речь идёт о попытке передать информацию используя свойство нелокальности в запутанной паре фотонов. В принципе, существует «no-communication», говорящая что это невозможно, но мне сложно оценить, насколько она применима к данному примеру.


      1. bull1251 Автор
        19.02.2018 22:13

        Когеренция теряется ещё в даун-конверторах.

        А где можно найти информацию, что это действительно так? Ведь подобные опыты уже проводились и интерференция на экране сохранялась. Я взял информацию исходя из проведенных опытов и под конец предложил свой вариант эксперимента.


        1. LSDtrip
          20.02.2018 12:31

          Весь смысл квантовой неопределенности в том, что для того чтобы «посмотреть на состояние системы» мы безвозвратно её меняем, поэтому получается ожидаемый результат. Этим в данной схеме как раз и занимаются даун-конверторы, которые смотрят на наши фотоны и тем самым вторгаются в систему. Энергию фотона и его траекторию нельзя измерить, не изменив их.
          Также и квантовая запутанность является следствием общего события в прошлом для двух частиц, таким образом измерив одну из них, можно точно сказать параметры другой, так как точно известно, сколько частиц и с какими параметрами является в следствии этого события в прошлом. Так что быстрее света всё равно ничего не передашь в рамках этой теории.


      1. Gibboustooth
        19.02.2018 22:53

        А разве с даун-конвертерами не делали двущелевых экспериментов с отложенным выбором и не получали интеренференционной картины?


        1. mihaild
          20.02.2018 02:26

          Получали. Подробнее arxiv.org/pdf/quant-ph/9903047.pdf
          Предлагаемая схема примерно эквивалентна fig. 1 (только BSA и BSB всегда отражают, D3 и D4 соответственно вообще нет, а двигаем BS).
          Проблема очень простая: в этой схеме мы на экране не увидим интерференционную картину вообще. Если BS стоит, то случаи, когда сработал D1, дают интерференционную картину (fig. 3), как и случаи, когда сработал D2 (fig. 4). Но, как хорошо видно, эти картины как раз друг друга компенсируют — если у нас есть информация только с D0, мы никакой интерференции не увидим, ее можно извлечь только последующей обработкой.
          (а если BS не ставить, то вообще нельзя)


          1. Gibboustooth
            20.02.2018 02:43

            Да, я примерно это написал ниже — тот эксперимент принципиально отличается от того, что предлагает автор.

            Но лично у меня вопросы остаются. Предположим, мы воообще уберем детекторы 3 и 4 и соответствующие зеркала. Оставим только дектор 0 и квантовый стиратель. Увидим ли мы в таком случае интерференционную картину? А если мы отнесём квантовый стиратель на Луну? А если мы иногда будем ставить на пути следования фотонов к стирателю детекторы?


            1. mihaild
              20.02.2018 02:57

              Какую конкретно часть установки вы предлагаете уносить? Предлагаю говорить об установке на fig. 1 из статьи, там меньше всего ненужных деталей:)
              Интерференционную картину на D0 мы не увидим ни в каком случае. Мы можем из холостых фотонов получить информацию, позволяющую разбить картину в D0 на две интерференционных. А можем из них же вытащить информацию о том, был фотон из A или из B. Причем выбирать, какую именно информацию будем вытаскивать, можем уже получив сигнал на D0.


              1. Gibboustooth
                20.02.2018 03:26

                Т.е. если мы заменим разделители BSA и BSB на обычные зеркала, картина на D0 не изменится — как не было там никакой интерференции, так и не будет?


                1. mihaild
                  20.02.2018 03:28

                  Нет конечно. Никакие операции с холостыми фотонами не изменят картину на D0.


                  1. Gibboustooth
                    20.02.2018 03:36

                    Спасибо за пояснения. После популярного описания этого эксперимента создавалось ощущение обратного (видимо, у автора статьи тоже).

                    Хотя результаты эксперимента от этого понятнее не стали — почему фотоны распределяются по детекторам таким образом, что мы можем «экстрадировать» из этих данных интерференционную картину на детекторе D0?


                    1. Tyusha
                      20.02.2018 12:35

                      Схема автора предполагает, что два даун-конвертора дают четыре запутанных фотона, а они дают две пары запутанных фотонов, а это, как говорится, две большие разницы. В этом есть, на мой взгляд, принципиальное отличие его схемы от того, что описано в упомянутой статье.


                      1. Gibboustooth
                        20.02.2018 13:48

                        Так в оригинальной схеме тоже 4 запутанных фотона. Суть-то в том, что никакая операция с холостыми фотонами ничего не поменяет на детекторе D0.


                        1. Tyusha
                          20.02.2018 14:02

                          Не сомневаюсь, что вы правы, но не могу сообразить, почему.


                        1. bull1251 Автор
                          21.02.2018 10:54

                          Суть в том, что в оригинальной схеме на детектор D0 попадает сумма результатов с интерференцией и без. Поэтому на D0 мы никакой интерференции не увидим, ее можно извлечь только последующей обработкой.
                          image
                          По этой причине я предложил в своем посте рассматривать интерференцию отдельно, не смешивая с результатом, где интерференция исключена.


                          1. mihaild
                            21.02.2018 13:51

                            Так а что с чем должно интерферировать? Учтите, что фотоны, по сцепленным с которым мы можем узнать, каким путем они прошли, не интерферируют.


                    1. mihaild
                      20.02.2018 14:20

                      Если очень грубо, то A и B излучают два «типа» фотонов каждый: A1, A2, B1, B2. При этом если мы будем смотреть только на A1 и B1, то увидим интерференционную картину, и если будем смотреть только на A2 и B2, то тоже увидим (только сдвинутую) — т.е. излучения от A1 и B1 складываются как волны (с учетом фазы), как и излучения от A2 и B2.
                      Излучатели устроены так, что A выпускает либо сигнальный фотон A1 и холостой фотон A2, либо наоборот (и B аналогично). Схема с зеркалами приводит к тому, что холостые фотоны типа A1 и B1 попадают в D1, а A2 и B2 — в D2. В итоге если мы будем учитывать на экране только фотоны, парные к которым попали в D1, то это окажутся ровно сигнальные фотоны типов A2 и B2, которые вместе дают интерференцию.

                      Тут куча вранья, на самом деле никаких «типов» фотонов нет, а есть амплитуды разных состояний, срабатывает только один из детекторов тоже из-за интерференции, и т.д. — но чтобы это всё описать правильно, нужно уже честно считать амплитуды разных результатов (которые считаются как сумма амплитуд разных способов получить результат — именно поэтому оказывается, что при наличии двух способов получить данный результат суммарная вероятность может оказаться меньше чем у каждого по отдельности).


                      1. Gibboustooth
                        20.02.2018 14:24

                        Схема с зеркалами приводит к тому, что холостые фотоны типа A1 и B1 попадают в D1, а A2 и B2 — в D2.

                        Да, но почему?


                        1. mihaild
                          20.02.2018 17:41

                          Собственно если переходить от болтологии к математике, то получается так (упрощенно, в реальном эксперименте всё было сложнее): для каждой точки X экрана есть два комплексных числа — A_X и B_X — амплитуды попадания в нее из A и B. Пара фотонов до попадания в экран соответственно находится в состоянии «сумма по всем X: (A_X|фотон из A, попадет в X> + B_X|фотон из B, попадет в T>)» (запись «U|V>» означает состояние V с амплитудой U).
                          Теперь мы поймали фотон в конкретной точке X. Система теперь находится в состоянии (A_X|фотон из A> + B_X|фотон из B>).
                          Если бы состояния «фотон из A» и «фотон из B» были бы неразличимы, то амплитуда попадания в X была бы A_X + B_X (важно: тут сумма комплексных чисел), вероятность обнаружения фотона в X была бы равна |A_X + B_X|^2 — опять складываются амплитуды (а не просто интенсивности), и мы бы видели интерференцию.
                          Но состояния «фотон из A» и «фотон из B» отличимы: убрав зеркало, мы по тому, сработает D1 или D2, можем их различить. Поэтому просто вероятность обнаружить фотон в X оказывается равна |A_X|^2 + |B_X|^2 — складываются не сами амплитуды, а квадраты их модулей. И никакой интерференции тут уже естественно нет (модуль амплитуды монотонно падает с ростом расстояния до щели).
                          Заметим, что в отсутствии зеркала, амплитуда состояния «фотон в D1» равна A_X, а амплитуда состояния «фотон в D2» равна B_X.
                          Теперь мы ставим зеркало — которое является преобразованием Адамара. Грубо говоря, оно из состояния |фотон до зеркала летит вверх> делает состояние (с точностью до множителя) |фотон после зеркала летит вверх> + |фотон после зеркала летит вниз>, а из состояния |фотон до зеркала летит вниз> делает состояние |фотон после зеркала летит вверх> — |фотон после зеркала летит вниз> (обратите внимание на разницу в знаках).
                          Заметим, что «фотон до зеркала летит вверх» = «фотон из B» (для удобства обозначим это состояние |B>), и «фотон до зеркала летит вниз» = «фотон из A» (обозначим как |A>).
                          Т.к. состояние «фотон после зеркала летит вверх» означает, что фотон попадет в D1, то обозначим это состояние как |D1>, а состояние «фотон после зеркала летит вниз» как |D2>.
                          Итого, к зеркалу фотон прилетает в состоянии A_X|A> + B_X|B>. После зеркала его состояние изменится на (A_X|D1> — A_X|D2>) + (B_X|D1> + B_X|D2>) = (A_X + B_X)|D1> + (B_X — A_X)|D2>. Теперь в D2 фотон оказывается с амплитудой B_X — A_X, а в D1 — с амплитудой A_X + B_X — и, соответственно, с вероятностями |B_X — A_X|^2 + |A_X + B_X|^2.
                          Из этого видно, что если фотон оказался в X, то вероятность обнаружить его в D1 пропорциональна |A_X + B_X|^2, а в D2 — |B_X — A_X|^2. Т.к. априорные вероятности детекторов одинаковы, то вероятность обнаружить фотон в точке X при условии, что он попал в D1, пропорциональна |A_X + B_X|^2. Опять же складываются амплитуды (а не модули), так что в каждом из этих случаев по отдельности мы увидим интерфенцию.
                          Упражнение: проверьте, что общая вероятность обнаружить фотон в X (т.е. |A_X + B_X|^2 + |A_X — B_X|^2) пропорциональна просто |A_X|^2 + |B_X|^2.


                          1. bull1251 Автор
                            20.02.2018 22:02

                            Мне кажется Вы рассматриваете этот эксперимент en.wikipedia.org/wiki/Delayed_choice_quantum_eraser
                            Там использовали два источника разной длины волны: image
                            И действительно, в каждом случае по отдельности можно было увидеть интерференцию. Но в общей картине интерференцию можно было извлечь только последующей обработкой. Это связано с тем, что использование двух источников разной длины волны создавало смещенную картину интерференции друг относительно друга, в сумме перекрывающих друг друга в общей картине:
                            image
                            В своем примере я предложил использовать только один источник, соответственно на экране может возникнуть только одна картина интерференции.


                            1. Gibboustooth
                              20.02.2018 23:12

                              Там использовали два источника разной длины волны

                              Откуда вы это взяли?


                              1. bull1251 Автор
                                22.02.2018 09:38

                                Ссори, в этом месте я действительно допустил ошибку. Судя по этой картинке
                                image
                                кажется, что результат R02 также отображает интерференцию, смещенный относительно интерференции R01. Такое смещение на проекционном экране возникает при изменение длины волны излучателя.
                                В действительности результат R02 отражает картину из двух полос, возникающее для фотонов, зарегистрированных детектором:
                                image
                                Но и в этом случае на проекционном экране возникает сумма всех результатов, позволяющая извлечь интерференцию только последующей обработкой.

                                Схематическое изображение эксперимента, использованное в посте, позволит получить такой же результат на экране, если на пути холостых фотонов будут установлены полупрозрачные зеркала (отметил стрелками):image
                                Так как половина выпущенных фотонов попадет на детекторы (3) и (4), а остальная половина попадет на детекторы (5) и (6) , то соответственно на экране возникнет сумма всех результатов, из которою извлечь интерференцию можно будет только последующей обработкой.
                                По этой причине, я предложил на пути холостых фотонов (3) и (4) устанавливать не полупрозрачные зеркала, а полностью отражающие зеркала. В этом случае фотоны попадут либо на детекторы (3) и (4) (давая картину из двух полос на экране) либо на квантовый стиратель (давая картину интерференции).
                                image
                                Никакого смешивания результатов на экране, только один результат!


                                1. Gibboustooth
                                  22.02.2018 10:20

                                  В уме наложите R01 на R02 (не забывайте, вы не можете их разделить без данных с детекторов (5) и (6)). Что получится? Тот же самый «колокол» что в R03 и R04 (только в 2 раза выше по интенсивности, но это не суть).

                                  Картину интерференции вы получите только если вы разделите данные детекторов (5) и (6). А сделать вы этого не можете, т.к. для этого вам надо передать эту информацию по традиционному каналу связи.


                            1. mihaild
                              20.02.2018 23:46

                              Я рассматриваю эксперимент из arxiv.org/pdf/quant-ph/9903047.pdf (упрощенный, изображенный там на fig 1).

                              Напишите явно, какие у вас амплитуды у чего. Если я правильно понимаю, что вы пытаетесь сделать, то ваш эксперимент аналогичен упрощенному из статьи, только у вас вместо пары атомов A и B, из которых излучает один, используются ДК.
                              (собственно в реальном эксперименте A и B даже не атомы, а разные участки одного кристалла)
                              Т.к. попадания на разные ДК отличимы, то для путей через разные ДК суммируются интенсивности, а не амплитуды.

                              Могу написать, что у меня получается, если вы с чем-то не согласны — поправьте.
                              В конечном итоге состояние перед попаданием на экран у вас должно быть зацепленным вида «сумма по всем точкам X: X_1(|фотоны пролетели через 1>?|контрольный попадет в X>) + X_2(|фотоны пролетели через 2>?|контрольный попадет в X>)».
                              С учетом того, что |фотоны пролетели через 1> и |фотоны пролетели через 2> отличимы, вероятность обнаружить контрольный фотон в точке X равна |X_1|^2 + |X_2|^2. Система после этого будет в состоянии X_1|фотоны пролетели через 1> + X_2|фотоны пролетели через 2>

                              (только пожалуйста не надо возвращаться к «фотон одновременно проходит по нескольким путям» — язык комплексных амплитуд вероятности строже и богаче)


                              1. bull1251 Автор
                                22.02.2018 09:50

                                Т.к. попадания на разные ДК отличимы...
                                Поясните пожалуйста что вы имеете ввиду?


                          1. mihaild
                            20.02.2018 23:48

                            Мне тут подсказывают:
                            >и, соответственно, с вероятностями |B_X — A_X|^2 + |A_X + B_X|^2
                            надо читать как
                            «и, соответственно, с вероятностями |B_X — A_X|^2 и |A_X + B_X|^2» (первое — вероятность обнаружить фотон в D2, второе — в D1)


                  1. AndrewRo
                    20.02.2018 13:13

                    Получается, если мы будем генерировать пары сцепленных фотонов, обстреливать одним из них экран, а второй поглощать и измерять, на экране по-прежнему будет интерференция? Не значит ли это, что мы можем узнать, через какую щель пройдёт первый фотон?


                    1. Tyusha
                      20.02.2018 13:37

                      Поглощать в какой момент, до щелей или после? Если до, то мне кажется, интерференция будет, т.к. запутанным ли он пададёт на экран или «распутанным» — не важно. Если же после щели, то интерференции быть не должно, тем более фотоны не различимы. Мне кажется, спутанность вообще ничего нового не привносит в вашем варианте.


                      1. AndrewRo
                        20.02.2018 13:44

                        Насколько я понимаю двухщелевой опыт, результаты будут зависеть от того, был ли фотон "измерен" до прохождения щели. Не является ли измерение запутанного с данным фотона аналогичным измерению самого фотона?


                        1. Tyusha
                          20.02.2018 13:58

                          Подождите, вообще надо договориться, что есть запутанность в данной ситуации? Обычно подразумевается, что строго определена некая суммарная величина спутанных частиц (обычно спин). Измерение спина вполне коммутирует с измерением координаты-импульса, а именно последнее важно для интерференции.


                          1. AndrewRo
                            20.02.2018 14:07

                            Ну, я подразумевал координаты-импульс. Ведь зная сумму и измерив координаты-импульс второй частицы, мы узнаём, в какую щель летит первая.


                            1. Tyusha
                              20.02.2018 14:22

                              del


                        1. mihaild
                          20.02.2018 14:26

                          Важно, «существует» ли возможность определить, через какую щель проходил фотон. Если да — то интерференции не будет видно.
                          Интерференцию мы можем увидеть после обработки, если воспользуемся информацией, которую нельзя получить одновременно с информацией о том, через какую щель фотон прошел.
                          Т.е. у нас один фотон давно попал на экран, у нас есть запутанный с ним. И два варианта, что с ним сделать — либо узнать, через какую щель прошел исходный (и попрощаться с интерференцией), либо узнать о нем что-то еще и с помощью этого восстановить интерференцию.
                          Важно, что «восстановление интерференции» — это постобработка изображения на экране, непосредственно на экране интерференционная картина магическим образом не появится.


                          1. AndrewRo
                            20.02.2018 15:34

                            Честно говоря, не понял ваших объяснений. Если фотон уже попал на экран, то какая разница, что там произошло или ещё не произошло со вторым? Мы ведь уже знаем точку его попадания и можем узнать, была ли интерференция.


                            1. mihaild
                              20.02.2018 15:40

                              На экране мы интерференцию не увидим. Экран дает нам информацию: «первый фотон попал в точку (0; 1)», «второй в (1.5, 0.7)» и т.д.
                              Но для каждого фотона есть еще парный фотон. И с ним можно делать разное:
                              1. Можно просто определить, что через левую щель прошли 1, 3, 4 и 10й фотоны
                              2. Можно поставить зеркало и посмотреть, куда упадут какие фотоны — и, если в один детектор попали скажем фотоны 1, 2, 7, 12, ..., то, оставив на экране только картинку от них, мы увидим интерференционную картину.


                1. Tyusha
                  20.02.2018 12:41

                  Я чёт не въезжаю, а почему на D0 вообще должна быть интерференция? Атомы A и B вроде как независимы в квантовом смысле.


                  1. mihaild
                    21.02.2018 00:10

                    Смотри третий вроде параграф («Рассеяние на кристалле») 8го тома Фейнмановских лекций.


  1. Rambalac
    19.02.2018 17:27
    +1

    Как замечательно в вашем мире. А в мире с шарообразной Землёй как?
    Я не заметил тег фантастика?


  1. Turbojihad
    19.02.2018 17:39
    +1

    фотоны достигают наблюдателя в тот же момент времени, как только они излучаются далекими звездами.

    Получается скорость света бесконечна, да?


    1. Gibboustooth
      19.02.2018 22:58

      Не очень понятно, как в СО фотона можно говорить о скорости, если нет понятия времени. Кстати, вот здесь статья с рейтингом +29, которая говорит о том же самом. А тут почему-то автора заминусовали.


    1. SandroSmith
      20.02.2018 00:06

      Нет. Просто с точки зрения фотона мы находимся не за миллионы световых лет, а тут-же, рядом.


  1. vconst
    19.02.2018 17:44
    +4

    Из песочницы
    Модераторы развлекаются?


    1. Boomburum
      19.02.2018 19:38
      +4

      Не знаем, как бороться с тем стереотипом, что из песочницы якобы должны выходить только шедевральные посты ) К сожалению, это не кузница золотых постов — это лишь барьер, который должен преодолеть любой новичок, чтобы стать участником сайта. Он должен приложить усилия, показать своё желание стать частью сообщества, чему-то учиться, но на то он и новичок — может чего-то не знать, неправильно оформлять материалы и тд. Если откровенный шлак мы отклоняем, а оформление ещё можем поправить, то все остальные дискуссии уже на совести сообщества )


      1. vconst
        19.02.2018 20:19

        Мне кажется, если очевидно, что статья получит резко отрицательную реакцию сообщества, автору от этого только хуже будет, чем если бы она зависла в песочнице.


        1. ploop
          19.02.2018 21:10
          +2

          Почему отрицательную? Я, например, с удовольствием почитал. Как научную фантастику конечно, но с удовольствием. И с таким же удовольствием читаю комментарии. Чего не скажешь о статьях про 100500й стартап на блокчейне.


          1. vconst
            19.02.2018 21:18
            +2

            Ну… Это конечно лучше, чем очередная блютусная мультиварка, согласен.


        1. Boomburum
          20.02.2018 00:38
          -1

          Ну вот есть человек, у которого в голове долгое время варилась некая теория, пусть и с ошибками в расчётах или по незнанию каких-то законов — он её опубликовал, аудитория помогла найти ошибку в расчётах, завязалась дискуссия. По-моему это наоборот классный процесс и тут не надо злиться и тд ) Всё равно что вы токарь и к вам пришёл стажёр — ему сначала надо научиться, пусть и через собственные ошибки. В данном случае стажёр ударил молотком по пальцу — в следующий раз будет осторожней :)


          1. Gibboustooth
            20.02.2018 00:53

            он её опубликовал, аудитория помогла найти ошибку в расчётах, завязалась дискуссия


            Вот только дискуссия быстро перемещается в карму автора, что неприятно и демотивирует писать подобные статьи.


            1. Boomburum
              20.02.2018 01:03

              Здесь, опять же, многое зависит от аудитории. Никто и ничто не мешает относиться к новичкам более лояльно :)


              1. Gibboustooth
                20.02.2018 01:31

                Теоретически — ничего не мешает. А по факту мы имеем то, что имеем :)


                1. Tyusha
                  20.02.2018 12:47

                  Несмотря на моё несогласие, проставилась плюсами.


                  1. Shkaff
                    20.02.2018 13:28

                    Учитывая качество большинства научных статей на гиктаймс в последнее время, эта просто шедевр.


                    1. Tyusha
                      20.02.2018 13:40

                      Не научные, а о науке.


                      1. Shkaff
                        20.02.2018 13:50

                        Я не был уверен, какое из написаных мною слов должно звучать наиболее саркастичным. Кажется, с "научным" не сложилось.


          1. vconst
            20.02.2018 05:00

            Вот я как раз на эту тему вспомнил о статье «полчаса телефонного разговора с физиком», навскидку не нашёл, но она тут есть. Там физику звонили по скайпу всякие фрики от науки, показывали свои пенопластовые модели вселенной или атомов из говна и палок из желудей и спичек. Но профессиональный ученый быстро объяснял, что для подробных теорий надо знать много сложной математики и без неё все «работает» только в научно-попсовых статьях. Фрики злились…


      1. usblexus
        20.02.2018 00:35

        Простите, но это не так, во первых откровенный шлак таки нет нет да проскакивает, во вторых напишу на собственном примере — года 1.5 -2 назад постил статью из своего опыта почему нельзя использовать тестовые задачи из интернета для набора специалистов, была отклонена дважды без объяснения причин, может я через чур субъективен но я не вижу там никакого шлака. Сейчас уже статья стала неактуальна т.к. софтверные компании начали отказываться от подобной практики


        1. Boomburum
          20.02.2018 00:39

          За 1.5-2 года модерация во многом поменялась, в том числе требования к ней. Можете попробовать ещё раз что-то написать :)


    1. Tyusha
      19.02.2018 20:01

      Не гони на НЛО.


  1. artemev
    19.02.2018 18:07
    +1

    ровно на столько, сколько потребуется скорости света, чтобы достичь красную планету

    То есть фактически получить информацию можно за то же время, как если бы мы передавали ее обычным способом? Зачем тогда городить этот огород?


    1. yuramalkoff
      19.02.2018 18:30

      Можно например, марсоходами вживую рулить.


      1. artskep
        19.02.2018 20:53

        Нельзя. Фотоны все равно будут лететь со скоростью света.
        В статье весьма прямо говорится о том, что задержка никуда не девается.

        После этого можно начать поочередно анализировать записанную информацию, чтобы получить передаваемую информацию. Для примера, если удаленный передатчик находится на Марсе, то анализ информации необходимо начинать с опозданием на десять-двадцать минут (ровно на столько, сколько потребуется скорости света, чтобы достичь красную планету).


        1. yuramalkoff
          19.02.2018 20:59

          Несмотря на то, что считывание информации идет с отставанием в десятки минут, полученная информация будет соответствовать тому, что передается с Марса в текущий момент времени.

          Я понял так, что задержка нужна для появления картинки интерференции, а дальше информация в реальном времени поступает.


          1. SandroSmith
            20.02.2018 00:08

            Ну так и по обычному каналу связи после задержки картинка в реальном времени ;)


        1. Turbojihad
          19.02.2018 21:08

          В статье прямо говорится о том что «фотоны достигают наблюдателя в тот же момент времени, как только они излучаются далекими звездами.»


          1. artskep
            19.02.2018 21:14

            Там уточняется, что с точки зрения ИСО фотона (что, вообще-то не совсем правильно применять с точки зрения СТО, т.к. это ее предельные условия)

            Но с ИСО частиц света, фотоны достигают наблюдателя в тот же момент времени, как только они излучаются далекими звездами.

            Я, конечно, рад за фотоны, но информацию хочется получать в той ИСО, в которой я нахожусь. А в этой ИСО фотоны будут лететь со скоростью света, и от ближайшей звезды (кроме Солнца) годами.


        1. bull1251 Автор
          19.02.2018 21:12

          Почему же нельзя? На Марсе информация кодируется в текущий момент времени, используя фотоны, выпущенные 10-20 минут назад. Поэтому мы анализируем запись с экрана с отставанием на 10-20 минут и получаем информацию, передаваемую с Марса в настоящее время. По традиционным каналам связи мы можем получить эту информацию только на 10-20 минут позже.


          1. Tyusha
            19.02.2018 21:21
            +2

            Послушайте, я вам уже указала на ошибку в вашей схеме на самом верху комментариев.


            Кроме того мгновенная передача информации нарушает принцип причинности. Мне кажется, примеры излишни, литературы навалом.


            1. Throwable
              20.02.2018 17:15

              Принцип причинности может появляться в локальных теориях как следствие более общих принципов (напр. самосогласованности). В нелокальных теориях он совершенно не обязан соблюдаться. Подобные сюжеты любят обыгрывать в научной фантастике, которые фактически сводятся либо к альтернативным реальностям, либо к самосогласованности.


          1. artskep
            19.02.2018 21:24

            По традиционным каналам связи мы можем получить эту информацию только на 10-20 минут позже

            Почему?
            Если мы дадим знать передатчику на Марсе когда передать информацию — она передастся как раз с задержкой в одном направлении.
            Очевидно, что с марсоходами и дальними зондами планируются сессии связи.

            Просто посмотрите на последнюю картинку, прикиньте где там свет идет долго и подумайте — есть ли какая-то разница, если вместо полупрозрачного зеркала просто поставили бы шторку (то биш получили бы амплитудно модулированный сигнал — классика жанра)?


            1. bull1251 Автор
              19.02.2018 21:40

              Фотоны передаются не с Марса на Земля, а наоборот с Земли на Марс. Т.е. на Земле картинка с экрана имеется за 10-20 минут до того, как на Марсе закодируют информацию. Поэтому результаты с экрана берут не сразу, а выжидают этот же интервал времени. Иначе мы получим информацию до того, как на Марсе закодируют информацию, или говоря простым языком, попытаемся заглянуть в будущее (этот момент описан в конце статьи).


              1. AllexIn
                20.02.2018 09:38

                Чем это практически отличается от ожидания прилета информации на световой скорости?


                1. Tyusha
                  20.02.2018 12:50

                  10-20 минут это в концепции автора, выражаясь компьютерным языком — латентность — время пока раскочегаривают сверхсветовой канал связи. «Лучше день потерять, зато потом за пять минут долететь.»


                  1. AllexIn
                    20.02.2018 13:20

                    А преимущество в чём?
                    Что мы вынуждены ждать, пока пройдет 10 минут в случае метода автора. Что мы вынуждены ждать 10 минут, пока сигнал дойдет…
                    Разницы никакой — результат нашей передачи будет через 10 минут.


                    1. Olga_Voronova
                      20.02.2018 13:36

                      Да, если подать сигнал SOS то разницы не какой нет. А если скажем управлять марсоходом в реальном времени то разница огромная. (в случае если автор прав конечно)
                      Это как… первые десять минут мы прокладываем кабель, а потом пакеты начинают бегать с пингом в 1мс в обе стороны.


                      1. AllexIn
                        20.02.2018 13:45

                        Нет же. Мы должны считать фотон через 10 минут. Каждый, а не только стартовые. ТО что он пришел сразу — ничего не меняет. Данные мы сможем забрать только через 10 минут.


          1. alexzzam
            20.02.2018 11:28

            Насколько я понял, вы утверждаете, что в эти 10-20 минут информация уже записана на флешку, но читать её нельзя, поскольку она ещё изменится, когда закончится период задержки? Но ведь именно этот эффект никогда и никем не наблюдался (afaik), что делает его доказательством ошибки в рассуждениях.
            А, кстати, если писать не на флешку, а за пять минут быстрым автоматическим резцом вырезать на металлической пластине, она по этой версии тоже изменится?


    1. artskep
      19.02.2018 20:46

      Если бы вместо двигающегося полупрозрачного зеркала предложили бы обычный затвор, а вместо сложной системы с разными квантовыми выдумками тупо фотодетектор — никого бы это не удивило.
      А так типа наука, квантмех и прочие умные слова, за которыми можно аккуратно замаскировать простой факт, что задержка никуда не девается.


  1. hssergey
    19.02.2018 18:28

    Далее, записанную информацию необходимо отложить до момента, пока холостые фотоны достигнут своего местоназначения. После этого можно начать поочередно анализировать записанную информацию, чтобы получить передаваемую информацию. Для примера, если удаленный передатчик находится на Марсе, то анализ информации необходимо начинать с опозданием на десять-двадцать минут (ровно на столько, сколько потребуется скорости света, чтобы достичь красную планету). Несмотря на то, что считывание информации идет с отставанием в десятки минут, полученная информация будет соответствовать тому, что передается с Марса в текущий момент времени.


    Ну а какой смысл в том, что информация будет соответствовать тому, что передается в текущий момент времени, если воспользоваться ей мы все равно сможем только через время, которое требуется на передачу информации «обычным» путем (по радиоканалу или еще как)? То есть для Марса через 21 минуту, для Альфа Центавры через 4.7 лет и так далее. Получается точно такая же проблема невозможности мгновенной передачи информации.

    Возможно что-то выйдет с попытками передать информацию через квантовую сцепленность, но и то я сильно сомневаюсь, что могут существовать такие «частичные» измерения, которые не «распутывают» частицы.


    1. iBat
      19.02.2018 20:52

      Если описанные автором опыты имеют место быть, то вроде бы ничто не мешает организовать двухстороннюю связь с Альфой Центавра. Земля отправляет свои фотоны, Центавр — свои. Когда те и другие достигают цели — получаем full duplex. Но чет меня терзают смутные сомнения.


      1. artskep
        19.02.2018 21:08

        Для этого надо лет за 5 договориться о том чего собираемся друг другу передавать.
        И в этом заключается главный «ой все» подобных систем, т.к. информация это мера неопределенности (грубо говоря), а если мы и так знаем чего получать, то канал связи не нужен :-)


        1. fedorro
          20.02.2018 12:22

          Передаем по этому мгновенному каналу, который уже на всякий случай установлен, заголовок сообщения, «Фотки заката 3х солнц», а потом сами данные. Но это всё как то очень просто выглядит для такой глобальной проблемы, над которой бьются уже не одно поколение учёных.


  1. Nuwen
    19.02.2018 18:39

    Далее опыт повторили еще раз, с включенными детекторами, но без записи данных о траектории движения фотонов. Несмотря на то, что опыт полностью повторяет предыдущий, за исключением возможности получения информации, через некоторое время на экране вновь образовалась интерференционная картина из светлых и темных полос.
    Вот здесь самый шаткий момент. Я никогда не понимал — действительно ли волновая функция схлопывается в момент наблюдения, либо в момент — когда фотон, находящийся в суперпозиции взаимодействует с любой другой частицей, которая в свою очередь уже может передать информацию о взаимодействии по цепочке в мозг наблюдателя; и неважно — передаёт ли нет — важен сам факт взаимодействия?


    1. AndrewRo
      19.02.2018 18:53

      Естественно, просто при взаимодействии. Первый вариант — это какой-то лютейший антропоцентризм.


      1. Nuwen
        19.02.2018 18:57

        А как же тогда парадокс Вигнера? Кроме того, никакого антропоцентризма не будет, если использовать интерпретацию Эверетта — на самом деле никакого схлапывания волновой функции не происходит, а наблюдатель видит определённый результат, потому что сам находится в точно такой же суперпозиции как и наблюдаемый объект.


        1. Zenitchik
          19.02.2018 21:38

          Под наблюдением в квантовой механике понимают взаимодействие фотона с датчиком, а не считывание человеком результата измерения.


    1. muhaa
      19.02.2018 19:19

      Обычно неформально считают, что сама макроскопическая действительность, в которой каждый объект уже определился где он находится, и изменить своего решения не может и является тем самым наблюдателем, который приводит к коллапсу вектора состояния микроскопической системы. При этом четкой границы между этой самой макроскопической действительностью и квантовым миром нет.
      ИМХО, нам хочется, чтобы мир состоял из мелких реально существующих частей, поведение которых давало бы поведение макроскопической реальности. Но согласно КМ получается, что реально существует макроскопическая действительность (реальность — по нашу сторону, а не в микромире), а если эту макро-действительность рассматривать очень детально, то мы видим множество возможных альтернативных микроскопических причин для наблюдаемой макро-картины.


  1. muhaa
    19.02.2018 18:56
    +5

    Если в голову пришла гениальная идея по физике, есть стандартный путь: опубликовать ее на dxdy, получить раздраженные разъяснения, закончить в разделе «Пургаторий». Чем виноват бедный geektimes?


    1. vconst
      19.02.2018 20:12

      Пургаторий
      БГЫГЫК :)


    1. vconst
      19.02.2018 20:15

      Здесь была статья, «полчаса телефонного разговора с физиком», очень интересная. Как раз про такие темы.


  1. HappyLynx
    19.02.2018 19:53

    Тот случай, когда сразу понимаешь, что мысленный эксперимент с изъяном, но, к сожалению, знаний не хватает, чтобы осмысленно найти противоречие.

    В качестве контр-эксперимента проведем следующую модификацию приведенного в статье мысленного эксперимента, в которой возникнет парадокс:
    // как в исходном эксперименте
    1. Создаем пучек фотонов на Марсе, делим его и прогоняем через ДК для наблюдения интерференционной картины.
    // модификация
    2. Отражаем «холостые» фотоны не на Землю, а, скажем, на Венеру, а уже с нее отражаем их на Землю.
    3. В момент наблюдения интерференционной картины на Марсе отправляем отдельным лазером прямо на Землю информацию о том, что видим.
    4. Так как сигнал Марс-Земля придет раньше, чем «холостые» фотоны Марс-Венера-Земля, то ничто не мешает провзаимодействовать с ними на Земле так, чтобы результат противоречил принятому сигналу. Получаем парадокс, а физика их жуть как не любит.

    И вообще, не стоит шутки шутить с причинно-следственными связями. Всегда вылезет боком.


  1. Tyusha
    19.02.2018 19:58

    Вы встали на скользкий путь деления на ноль. А оно позволяет чудеса.


    1. denis64
      19.02.2018 20:15

      Господа, подскажите пожалуйста ссылку на статью о делении на ноль на geektimes. В статье рассказывалось о математической теории с наличием двух нулей, отрицательного и положительного. Никак не могу найти ту статью, может уже удалена. Заранее благодарю.


      1. andreyvlv
        19.02.2018 23:43

        Не эта статья? habrahabr.ru/post/249431


        1. denis64
          20.02.2018 22:05

          Да, она. Спасибо.


  1. Oliksolik
    19.02.2018 20:21

    Новый вид криокамеры — пускаем человека со скоростью света и пусть себе крутиться вокруг солнечной системы, а когда нужно будет «разморозить» тормозим капсулу.


    1. Tyusha
      19.02.2018 21:25

      Ну… так оно в общем-то и есть. Дело за малым: разогнать и обеспечить круговую орбиту этой центрифуге с невиданными перегрузками.


      1. artskep
        19.02.2018 21:35

        Ну, и еще как-то затормозить/завернуть/вернуть на Землю. Ибо даже третья космическая выглядит с точки зрения ТО жалко.

        Хотя можно и не возвращать: Пол Андерсон «Тау Ноль» :-)


    1. denis64
      19.02.2018 21:41

      Не обязательно покидать землю. Световых скоростей можно добиться слепив на кухне, на коленке, систему из веревок и роликовых блоков, с (если мне не изменяет память) 31 блоком при начальной скорости 1 м/с


      1. artemev
        19.02.2018 21:44

        Походу крепкая должна быть веревка :)


      1. DmitriyN
        19.02.2018 21:46
        +1

        Ага, только перед этим надо обязательно взять лист бумаги и сложить его пополам 20 раз.


  1. mecotrade
    19.02.2018 21:28

    Коллеги, ваш стёб, конречно, весело читать, однако ведёте себя вы совершенно неспортивно. По следующим причинам:
    1) Причина прецедентная. Во время оно Альберт Германович Эйнштейн был крайне несогласен с Нильсом Христиановичем Бором относительно базовых аспектов квантовой механики. Но вместо остёбывания научного оппонета он предпочел сформулировать свои претензии в виде строгих утверждений. Большую часть из которых Нильс Христофорович опроверг, а из меньшей родился, например, парадокс ЭПР. Что родится из вашего стёба — в лучшем случае ничего.
    2) Идея конечности скорости света была предложена Анри Леоновичем Пуанкаре и развита уже известным нам Альбетром Германовичем Эйнштейном. При этом изначально речь шла об электромагнитном поле, а затем с лёгкой руки Альберта Германовича была распространена на все массивные и безмассовые материальные объекты. Важно подчеркнуть, что об информации речи не было, да и сейчас, если вы откроете любую книжку по СТО/ОТО, информация не присутствует в математическом формализме. Поэтому когда утверждается о том, что СТО/ОТО запрещает передачу информации со скоростью выше скорости света, делается неявное предположение, что иного способа, кроме как посадить информацию на пучок фотонов/электронов, и т.д. (нужное подчеркнуть) не существует
    3) Предположение кажется вполне разумным, однако я хочу обратить внимание, что это всего лишь предположение, и на сегодняшний момент не существует математически (или физически) строго результата, доказывающего, что по-другому передавать информацию невозможно. Если я не прав — поделитесь ссылкой.
    4) Хорошо известная логическая максима утверждает, что отрицательные утверждения невозможно доказать (если только они не сводятся к положительным). Так что автор статьи имеет полное право пытаться конструктивно доказать возможность построения сверхсветовой связи. Если получится, то, как говорится, in your face, если же нет — это не значит ровным счётом ничего.
    5) Ну и напоследок — вспомним, что история науки показывает, что любая универсальная догма рано или поздно обнаруживает границы своего применения. Вспомним ту же плоскую Землю. Поверьте, что учителя учителя учителя (n раз повторить) вашего учителя/преподавателя, который рассказал вам о непреодолимости скорости света учили, что Земля-таки плоская. А оно эвон как вышло…


    1. artskep
      19.02.2018 21:40

      Поэтому когда утверждается о том, что СТО/ОТО запрещает передачу информации со скоростью выше скорости света, делается неявное предположение, что иного способа, кроме как посадить информацию на пучок фотонов/электронов, и т.д. (нужное подчеркнуть) не существует

      Ок, в данном случае автор предлагает передачу информации все-таки пучком фотонов. Значит все, что писали ученые мужи до этого применимо — можно закрывать дискуссию.

      А теперь вопрос вам: вы так фамильярно назвали здесь присутствующих «коллегами» — позвольте поинтересоваться вашими заслугами тогда? Потому что вы, извините, судя по вашему комментарию и комментариям в профиле, про ОТО/СТО несете чушь.


      1. mecotrade
        19.02.2018 22:44

        К.ф.-н.м, PhD в области теоретической физике, диссертация посвящена непертурбативным вычислениям в суперсимметричных теориях Янга-Миллса и связи оных теорий с теорией суперструн. Можно было бы померятся индексами цитирования и прочее, но, судя по вашей способности делать выводы из комментариев, меряться вам особо нечем.
        Так что да, когда разговор идёт о физике называя кого-то «коллега» я делаю комплемент. Видимо, в вашем случае я это сделал напрасно. За что приношу свои извинения.


        1. muhaa
          19.02.2018 23:24

          Изложите вашу идею про сверхсветовую передачу информации физикам на каком-нибудь специализированном физическом форуме, а мы посмеемся. Здесь же мы можем возразить вам только простой логикой: если сверхсветовая передача информации возможна, то можно построить сверхсветовой передатчик и передать информацию в будущее (в определенной системе отсчета). Тогда это либо нарушит причинность (можно будет узнать собственное будущее обменами сигналов в разных СО), либо нарушит принцип относительности СТО, который очень сложно выкинуть из физики (вам легче представить насколько сложно, как специалисту).


          1. ploop
            20.02.2018 00:02

            В прошлое, а не будущее, не?
            В будущее мы её передаём постоянно, даже прямо сейчас.


            1. muhaa
              20.02.2018 00:59

              Точно, в прошлое. В смысле, что можно будет поговорить с собой будущим. Или принцип относительности не верен.


    1. Tyusha
      19.02.2018 21:50

      1. Спору нет. Ошибки предложенной схемы автора я предъявила выше.

      2. СТО не утверждает максимальности скорости света. Это является неизбежным следствием одинаковости скорости света в разных системах отсчёта. А это экспериментальный факт. Так что ничего Альберт не выдумал, а вывел это из опыта, опровержение которому за 115 лет не найдено.

      3. СТО не опровергнута, а значит сверхсветовая связь будет нарушать причинность. Стоит заметить, что современная физика нигде не требует соблюдения причинности! Поэтому её среди постулатов физики нет (да-да, друзья, это так). Однако от соблюдения причинности пока человек отказаться не может, т. к. это противоречит нашей логике. Хотите сверхсветовую связь — сначала придумайте, как быть с причинностью. А так да, такую передачу теоретически ничего не запрещает, хотя и не видно, как это может быть реализовано не только в рамках не только открытой экспериментального физики, но и в теоретических построениях.


      1. Oliksolik
        19.02.2018 22:44

        А как сверхсветовая связь может нарушать причинность? Послали сигнал с Марса и тут же его приняли на земле, что не так? Или вы о чём то другом? Или я)


        1. Tyusha
          19.02.2018 23:53

          Если вы передаёт сигнал мгновенно в какой-то системе отсчёта, то существуют системы отсчёта, где отправка сигнала происходит позже его получения. Что и есть нарушение причинности.


          1. dkukushkin
            20.02.2018 05:05

            Могли бы вы объяснить на примере о таких системах отсчета (где отправка сигнала происходит позже его получения)?


            1. HappyLynx
              20.02.2018 09:47

              См. мой комментарий выше:
              geektimes.ru/post/298309/#comment_10619081


          1. Oliksolik
            20.02.2018 11:09

            эм… совсем запутался. Сигнал то ещё не создан как его можно принять раньше? Это уже какое то путешествие во времени.


            1. vedenin1980
              20.02.2018 11:47
              +1

              Ну смотрите отправлял корабль за 100 световых лет со скоростью близкой к скорости света. Сразу после запуска включаем мгновенную видеосвязь.

              Поскольку относительно корабля Земля тоже летит со скоростью света время на Земле относительно корабля идет замедленно (обратное тоже верно на корабля относительно Земли тоже), то есть к моменту прилету корабля к Звезде видеосвязи на стороне Земли пройдет, скажем 1 день.

              Через 100 лет на Земле включаем новую мгновенную видеосвязь с кораблем. В результате к моменту прилету корабля к точки назначения, у нас две видеосвязи — одна через 1 день после запуска, вторая через 100 лет после запуска.

              Поворачиваем два экрана к друг другу и передаем информацию о своем будущем, которое можно будет изменить в прошлом (банально поговорив со своим внуком, потом не заводить детей, то есть внук окажется не существующим, но повлиявшим на ваше прошлое). А это уже временной парадокс или «нарушение принципа причинности», когда событие в будущем влияет на прошлое, да еще и возможно так чтобы события в будущем не произошло.

              Так как физика не любит парадоксов — в ОТО мгновенной передачи быть не может. Можно оказаться от относительности времени и придумать некоторое абсолютное время, но для этого придется полностью выкинуть ОТО и заменить ее совсем другой теорией.


              1. Oliksolik
                20.02.2018 12:19

                не понял) а если включить сразу связь и смотреть на экран весь полёт с Земли? как там и тут мы будем смотреть все 100 лет и ничего не нарушиться. Иначе мы должны видеть супер ускоренное видео


                1. vedenin1980
                  20.02.2018 12:27

                  Проблема по ОТО в том что время на ракете замедляется относительно Земли за счет скорости света. Надеюсь вы про это знаете? Но за счет относительности отчета, в точки отсчета корабля Земля тоже движется со скоростью света и там тоже время замедляется.

                  Мы получаем парадокс когда с каждой стороны видеосвязи мы должны наблюдать замедление времени, что невозможно, либо должно приводить к возможности передачи информации из будущего в прошлое (как я описал выше). И то и другое плохо с точки зрения физики, хуже чем нарушение законов сохранения.


                  1. Oliksolik
                    20.02.2018 13:59

                    спасибо, примерно понял) Оказывается причина непонимания в замедление времени. Это же получается что все хим. процессы, распад атомов. и тд тоже замедляются. Интересно, как борт коробя будет взаимодействовать с окружающим пространством ведь у них будет как бы«другая» химия.


                    1. vedenin1980
                      20.02.2018 14:38

                      Химия то ладно, все равно на таких скоростях не успеют провзаимодействовать до того как аннигилируют, там каждая песчинка как атомная бомба. Там даже с геометрией все запутано: Парадокс шеста и сарая


            1. alexzzam
              20.02.2018 11:47
              +1

              Так и есть. Только на физическом языке это называется не «путешествие во времени», а «нарушение принципа причинности». Не факт, что это невозможно, но пока похожего не наблюдалось.


      1. mecotrade
        19.02.2018 22:51

        СТО не утверждает максимальности скорости света. Это является неизбежным следствием одинаковости скорости света в разных системах отсчёта.
        Это и называется «СТО утверждает»
        Так что ничего Альберт не выдумал, а вывел это из опыта, опровержение которому за 115 лет не найдено.

        утверждения Альберта Германовича Эйнштейна (уважаемый человек, не стоит фамильярничать) относились к свету и материальным телам, с этим, собственно, никто не спорит. По поводу информации он не утверждал ничего.
        Стоит заметить, что современная физика нигде не требует соблюдения причинности! Поэтому её среди постулатов физики нет (да-да, друзья, это так). Однако от соблюдения причинности пока человек отказаться не может, т. к. это противоречит нашей логике.
        Иными словами это ваша религия. Имеете право, у нас многоконфессиональное государство. Но, не навязывайте свои религиозные взгляды окружающим. Как говаривал Дж. Карлин, keep thy religion to thyself


        1. BigBeaver
          19.02.2018 23:01

          По поводу информации он не утверждал ничего.
          Существование информации без носителя не показано.


          1. mecotrade
            19.02.2018 23:18

            Как не показано и несуществование информации без носителя. Поэтому желающие опровергнуть сверхсветовую передачу информации ссылаясь на СТО/ОТО должны вначале доказать несуществование информации без носителя. О чем я здесь и толкую.


            1. BigBeaver
              19.02.2018 23:23

              Доказывать обобщенное несуществование методологически не корректно. В случае же существования можно предъявить частный случай и этого будет достаточно.


              1. mecotrade
                19.02.2018 23:42

                Верно, поэтому отсылка к обеим Теориям Относительности никак и не помогает a priori отвергнуть возможность сверхсветовой передачи информации. С другой стороны, если автору статьи (или еще кому-нибудь) удастся конструктивно доказать такую возможность, т.е. предъявить (возможно, только теоретически) действующую схему, это будет неявно доказывать, что информация не обязана передаваться исключительно фотонами/электронами.


                1. Gibboustooth
                  19.02.2018 23:58

                  это будет неявно доказывать, что информация не обязана передаваться исключительно фотонами/электронами


                  Уже экспериментально показано, что информацию можно передавать изменением кривизны пространства-времени. В позапрошлом году на эту тему была куча статей всех степеней научности. И Нобелевку дали, само собой.


                  1. Tyusha
                    20.02.2018 12:05

                    Гравитационные волны распространяются опять-таки со скоростью света.


                1. Storm03
                  20.02.2018 00:50

                  Помню, в одной старой книжке читал, что если взять гильотинные ножницы, то можно подобрать параметры (скорость падения и угол), когда точка реза будет перемещаться быстрее скорости света. Можно ли в таком опыте утверждать, что точка реза — это нематериальная информация, перемещающаяся со скоростью, выше скорости света?


                  1. Zenitchik
                    20.02.2018 01:15

                    Точка реза не переносит никакой информации.


                    1. Storm03
                      20.02.2018 09:22

                      Это если лезвия идеально гладкие. А если на них нанесены «зазубринки» определённым способом, то широкий луч света, идущий через щель, будет ими «промодулирован».


                      1. Gibboustooth
                        20.02.2018 13:56

                        Представьте себе треугольник OAB, где О — это источник излучения, а A, B — две удаленные от него точки. Проведя лучом из точки А в точку В вы передаете информацию по каналу О -> А и по каналу O -> B. Со скоростью света, естественно. Но не по каналу A -> B.


                        1. Storm03
                          20.02.2018 16:02

                          Да, Вы правы.


                    1. Storm03
                      20.02.2018 09:29

                      Конечно, носитель информации — луч света, будет двигаться со своей любимой скоростью, но он будет нести информацию, закодированную быстрее скорости света?
                      Я выразился очень сумбурно, но, надеюсь, Вы понимаете, о чём я?


                      1. ploop
                        20.02.2018 09:45

                        Поливали когда-нибудь огород или что-то ещё из шланга? Двигать им из стороны в сторону можно быстрее скорости воды, вытекающей из него, и как струя потом ложится на землю тоже понятно.

                        Теперь мысленно попробуйте каким-нибудь образом промодулировать этот поток так, чтобы передать информацию быстрее? Максимум, при резком движении через «модулятор» (сетку например), вы получите много параллельных струек, которые движутся с той же скоростью.


                        1. Storm03
                          20.02.2018 16:01

                          Да, я уже понял, что фигню написал. Мы просто уплотним сигнал. А скорость передачи информации останется та же.


                          1. ploop
                            20.02.2018 16:20

                            Тут как ни крути, через обычный физический носитель (тот же свет) передать быстрее, чем движется сам носитель не выйдет.

                            А вот пофантазировать на тему всяких квантовых эффектов (что и сделал автор) довольно интересно.


                            1. mihaild
                              20.02.2018 16:24
                              +1

                              Вот почему всем интересно «фантазировать», а прочитать учебники и узнать, как на самом деле — неинтересно?(


                              1. ploop
                                20.02.2018 16:26

                                Вот почему всем интересно «фантазировать», а прочитать учебники и узнать, как на самом деле — неинтересно?(

                                Потому, что читали, и как на самом деле — известно. На данном этапе развития науки. И лишь фантазии могут раздвинуть эти рамки.


                                1. mihaild
                                  20.02.2018 17:47

                                  Ну вот я читал учебники, вы, возможно, тоже. Автор темы явно учебники не читал.
                                  А без этого не получится даже понять, на какие именно вопросы сейчас ищут ответы, на какие уже нашли, а какие вопросы оказались бессмысленными.


                                  1. ploop
                                    20.02.2018 19:37

                                    Автор темы явно учебники не читал.

                                    Оставим это на его совести, главное — тему такую затронул, что комменты просто взахлёб читаешь! :)


                  1. BigBeaver
                    20.02.2018 09:21

                    Это мой любимый пример сверхсветового движения. Солнечные зайчики еще подходят. Но, как верно заметил Зенитчик, информацию это не несет.


            1. vedenin1980
              20.02.2018 01:08

              Как не показано и несуществование информации без носителя. Поэтому желающие опровергнуть сверхсветовую передачу информации ссылаясь на СТО/ОТО должны вначале доказать несуществование информации без носителя. О чем я здесь и толкую.

              Представим, что мы отправили корабль от Земли со скорость 0.9 солнечной на борту, которого есть аппарат с мгновенной скоростью передачи (или намного более быстрой чем световая).
              Относительно Земли время на корабле замедлилось и там за день идет за неделю на Земли, но Земля относительно корабля тоже движется со скоростью света и там тоже день идет за неделю.

              Если передавать организовать скайп, то при таком разговоре по ОТО на Земле должны видеть замедления времени на корабле, а на корабле — замедление времени на Земле. Одновременно. Парадокс.
              Либо звонок по одной из систем уйдет в прошлое (например до запуска корабля) и можно например передумать и не запускать корабль. Тоже парадокс.

              Либо события одновременные в одной из систем, окажутся не одновременными с точки зрения другой системы, то есть можно играть на бирже зная курс через несколько секунд.


              1. mecotrade
                20.02.2018 01:43

                А потом этот корабль облетев всю галактику вернется на Землю, и брат-близнец, оставшийся на Земле, будет сильно старше своего брата-космонавта, поскольку время на корабле текло медленее. Но, с другой стороны, брат-космонавт должен быть старше брата-землянина, поскольку время на Земле текло медленнее.
                Как видите, парадокс возникает без всякой сверхсветовой связи.
                Между прочим, именно поэтому он называется "парадокс близнецов", а вовсе не потому, что один из братьев постареет сильнее, как ошибочно утверждает популярная литература


                1. arheops
                  20.02.2018 05:43

                  Нет, описанный вами парадокс будет только если он НЕ ЗАМЕДЛЯЯСЬ вернется в ту же точку, что пока невозможно.
                  При замедлении ваш парадокс исчезнет.


                1. vedenin1980
                  20.02.2018 10:06

                  Ну если вы, как утверждаете, к.ф.-н.м, PhD в области теоретической физике, то уж знать, что это парадокс прекрасно решается в рамках ОТО за счет ускорения. Вывод — вы просто тролите или придумали про свои научные степени.


                  1. wadeg
                    20.02.2018 17:05

                    Но ведь уважаемый vedenin1980 прав буквально в каждом слове. Парадокс близнецов является парадоксом только для начинающих любопытствующих. Ну и прочие комментарии mecotrade соответствует того же уровня познаниям, уж принцип причинности будущим физикам точно объясняют в подробностях. Так что минусы vedenin1980 непонятны.


                  1. mactep3230
                    20.02.2018 20:09

                    Если не придумал, то становится страшно. За теоретическую физику.


              1. ploop
                20.02.2018 08:32

                о есть можно играть на бирже зная курс через несколько секунд.

                Надо озвучить это биржевикам, сколько денег потечёт в науку… :)


        1. ildarz
          20.02.2018 12:46

          утверждения Альберта Германовича Эйнштейна (уважаемый человек, не стоит фамильярничать) относились к свету и материальным телам

          Скажем так — они относятся к любым физическим взаимодействиям, распространяющимся с конечной скоростью. Нет физических взаимодействий, которые распространялись бы с конечной скоростью, превышающей скорость света. Однако конечность скорости распространения физических взаимодействий в ТО постулируется на основе опыта. Т.е. если существуют взаимодействия, распространяющиеся с бесконечной скоростью, ТО это никак не противоречит, она просто их не описывает.


      1. mecotrade
        19.02.2018 22:58

        И кстати о причинности. Если с вазой упавшей на пол и разбившейся вследствии этого все понятно, то как быть с испускающим фотон атомом: переходит ли электрон на более низкий энергетический уровень вследствии испускания фотона, или наоборот, испускает фотон по причине перехода на низкий энергетический уровень. Как вы будете определять, что здесь причина, а что следствие. А если вы не можете это определить, какая польза от принципа причинности?


      1. wych-elm
        19.02.2018 23:02

        Вот все говорят о причинно-следственной связи, а что это такое с точки зрения физики? Причина и следствие в общем, это исключительно субъективные вещи, они конкретны в литературе, формальной логике (и логике вообще), но какое они имеют отношение к материальному миру? Фактически, все процессы описываемые математическим аппаратом в физике, имеют свойство обратимости. Основополагающим свойством причинно-следственной связи является ее направленность вперед во времени, т.е. это, по сути, воплощение т.н. «стрелы времени». Но разве существует какие-либо теории или эксперименты утверждающие и доказывающие единственное возможное направление этой стрелы? И если передача информации быстрее света ограничена только «нарушением причинно-следственной связи», то такое ли это незыблемое ограничение?


        1. Zenitchik
          19.02.2018 23:04

          Фактически, все процессы описываемые математическим аппаратом в физике, имеют свойство обратимости

          Второй закон термодинамики негодует.


          1. Tyusha
            20.02.2018 12:16

            Вы не совсем правы. Второй закон термодинамики запрещает, так сказать, обращение времени вспять. Но нарушение причинности, это не совсем то, оно «может» возникать и без обращения времени. Например замкнутые временеподобные петли вполне себе могут быть при условии существовании «кротовых нор». Это будет нарушением причинности, но не термодинамики.

            Заметьте, при путешествии через «кротовую нору» (или «червоточину», как их ещё называют) нигде локально не нарушает второй закон термодинамики. Однако, интересно, когда вернёмся в точку старта после такого путешествия. У нас должна быть исходная энтропия. Получается или по пути она нигде не должна была расти, что скорее всего нереализуемо. Или мы должны были по дороге излишнюю энтропию сбросить вовне, что выглядит более реалистичным (впрочем слово «реалистичность» здесь вряд ли уместно).


            1. Shkaff
              20.02.2018 12:58

              Я бы сказал, что причинность более строгий принцип в физике, нежели второй закон. Второй закон без проблем нарушается в квантах сплошь и рядом.
              И разве причинность нарушается в кротовых норах?


              1. Tyusha
                20.02.2018 13:49

                Через кротовую нору мы телепортируемся в другое место, т.е. перемещаемся быстрее света с точки зрения внешнего пространства. Поэтому, думаю, можно сконструировать ситуацию с движущимися кротовыми норами, когда причинность нарушается.


                1. Shkaff
                  20.02.2018 13:56

                  Хм, ну надо же учитывать замедление времени, все такое. Мгновенным перемещение будет только для попавшего в нору. С точки зрения наблюдателя он вообще никогда не пересечет горизонта норы (по крайней мере для решения Шварцшильда). А вообще, кротовые норы — стандартное решение ОТО, не представляю, каким бы образом там могла нарушаться причинность, тк причинность — основа ОТО.


                  1. Tyusha
                    20.02.2018 14:06

                    Может быть вы и правы насчёт замедления времени, сходу не сообразить.

                    А насчёт причинности в ОТО. Локальное соблюдение причинности, как я понимаю, не исключает глобальных топологий с замкнутыми временеподобными мировыми линиями.


                    1. Shkaff
                      20.02.2018 14:19

                      Тут вы правы насчет CTC, дело темное:)


                      Хотя как я понимаю, большинство в физике считают, что причинность и в этом случае должна сохраняться, только не очень ясно пока как.


        1. vibornoff
          20.02.2018 13:10

          Фактически, все процессы описываемые математическим аппаратом в физике, имеют свойство обратимости.
          Так то оно, конечно так, и, более того, вам даже ничто не запрещает двигаться вспять во времени, но ваша энтропия в этом случае будет уменьшаться, стирая информацию из будущего. Так что, даже если вы иногда и будете так делать, узнать об этом нет никакой возможности :-)


          1. wych-elm
            20.02.2018 16:20

            Не обязательно, проиллюстрирую свою точку зрения видоизмененным мемом «об украденном велосипеде»: «Однажды я обнаружил что кто-то двигал мою систему назад во времени. Мое количество энтропии уменьшилось. Но кто-бы это ни сделал, его количество энтропии значительно увеличилось. Общий уровень энтропии во Вселенной возрос, поэтому я остался спокоен.»


        1. Hardcoin
          20.02.2018 13:29

          Физика очень сильно опирается на наблюдение. Эксперимент — основа познания. Утверждать, что принцип причинности выполняется не всегда, можно, но без примера это особого значения не имеет.


    1. Turbojihad
      19.02.2018 21:51

      Во время оно Альберт Германович Эйнштейн был крайне несогласен с Нильсом Христиановичем Бором относительно базовых аспектов квантовой механики. Но вместо остёбывания научного оппонета он предпочел сформулировать свои претензии в виде строгих утверждений.

      А давно гиктаймс стал форумом физиков? Может быть у товарища bull1251 познания на уровне Нильса Бора, но большинство комментирующих вряд ли сравнивают себя с Эйнштейном.
      Такую статью еще бы на пикабу запостить, и удивляться потом комментариям.


    1. Darth_Biomech
      20.02.2018 16:19

      Поэтому когда утверждается о том, что СТО/ОТО запрещает передачу информации со скоростью выше скорости света, делается неявное предположение, что иного способа, кроме как посадить информацию на пучок фотонов/электронов, и т.д. (нужное подчеркнуть) не существует

      Информацию можно передавать хоть мысленными усилиями эльфов, суть от этого не меняется, ровно так же как и от того, насколько именно быстрее скорости света происходит передача — в итоге все равно есть возможность нарушения причинности и путешествий во времени.


  1. Num
    19.02.2018 22:48

    Итак, мы теперь знаем, что сигнальные и холостые фотоны одновременно достигают экрана и наблюдателя не зависимо от расстояния, так как для фотонов не существует времени и пространства.

    Вот примерно отсюда дальше можно не читать, потому как автор рассматривает мир из системы координат фотона, а потом начинает что-то конструировать с точки зрения внешнего наблюдателя.


    1. bull1251 Автор
      20.02.2018 16:31

      Спасибо, что указали неточность, исправил.


  1. Gibboustooth
    19.02.2018 23:52

    Вы не совсем верно описываете двущелевой эксперемент с отложенным выбором и квантовым стирателем, который был произведен в 1999 году. Отсюда у вас и получается мгновенная передача информации и нарушение причинности.

    В том эксперименте никаких зеркал никто не двигал, детекторы, обозначенные на вашей схеме 3, 4, 5, 6 работали одновременно, а вместо нарисованных у вас движущихся зеркал стояли полупрозрачные зеркала (т.е. фотон с вероятностью 50% летел в детектор 3 [или 4]), а с вероятностью 50% — в квантовый стиратель, к детекторам 5, 6. Затем, зная время прихода фотонов на каждый из детекторов, можно было разделить их «двойников». И получалось, что «убрав» фотоны, чьи двойники попали на детекторы 3, 4, мы получали интерференционную картину. Но сделать это можно только обладая информацией с детекторов 3, 4, 5, 6.

    Что будет, если двигать зеркала? Уверен, что информация о сдвиге зеркала на расстоянии 10 световых минут «придёт» к вам (в виде изменения паттернов фотонов на основном детекторе) через те самые 10 минут. А полученное в ходе эксперимента сообщение будет гласить
    Хорошая попытка, но нет. С любовью, принцип причинности.

    Возможно, я ошибаюсь, но как-то сомнительно, что ни один физик в мире не подумал о такой очевидной возможности найти нарушение одного из фундаментальнейших принципов в физике. Это ж 100% Нобелевка.


  1. dan939
    20.02.2018 00:34

    Подобное техническое решение позволило инженерами компании Toshiba построить 1984 году транзистор с плавающим затвором, ставшим основой для построения современных энергонезависимых микросхем памяти.


    а не поясните как в 1982 году в СССР выпускались микросхемы на транзисторах с плавающим затвором?


  1. smer44
    20.02.2018 00:39

    а только мне кажется что отражённый от зеркало фотон или даже прошедший через полупрозрачное стекло это другой фотон, который излучило зеркало или стекло?


    1. Gibboustooth
      20.02.2018 00:45

      Интерференция получается экспериментально, так что фотон, видимо, тот же самый.


    1. quwy
      20.02.2018 03:23

      Если считать фотон корпускулой, то отражение суть упругий отскок. Если волной, то поглощение и повторное излучение. Вам какой вариант больше нравится?


      1. Zenitchik
        20.02.2018 15:33

        Почему бы корпускуле не быть поглощённой (вступить в реакцию), а потом не выделиться вновь (в результате распада)?


    1. Shkaff
      20.02.2018 09:59

      Согласно принципу неразличимости, если два фотона находятся в одном состоянии — нельзя отличить один от другого. Так что если при отражении (переизлучении) фотон остался в том же состоянии, что и был до — это тот же самый фотон по правилам квантовой механики.


  1. leok
    20.02.2018 00:39

    Далее опыт повторили еще раз, с включенными детекторами, но без записи данных о траектории движения фотонов.

    Вот тут вы вводите людей в заблуждение. Запись должна быть не отключена, а принципиально невозможна.


  1. Nick_Shl
    20.02.2018 00:39

    А как устроены датчики и ловушки позволяющие засечь одиночный фотон проходящий через щель да ещё и не оказав никакого влияния на него?


  1. Rfvhu
    20.02.2018 00:39

    Автор в курсе про Парадокс Эйнштейна — Подольского — Розена?


  1. Occamlab
    20.02.2018 00:53

    Пётр Маковецкий, «Смотри в корень!»
    задачки 44 и 45
    n-t.ru/ri/mk/sk044.htm
    n-t.ru/ri/mk/sk045.htm


  1. OlegXD
    20.02.2018 01:01
    +1

    Людям свойственно ошибаться, заблуждаться и мечтать.
    Думаю, о подобном (скорости передачи данных выше скорости света) мечтают многие, что подтверждается большим количеством комментариев.
    На первый раз можно было бы оставить автора наедине с его рассуждениями, ошибками и заблуждениями (без минусов в карму), на мой взгляд. Прочитав опровержения и негативные комментарии, в следующий раз более тщательно будет прорабатывать свои же мысли.
    В дополнение к оспариванию такого способа могу предложить автору один вопрос и один мысленный эксперимент:


    1. Чем отличается с точки зрения скорости передачи данных Ваш прибор от обычного лазера, которым "отбивают" азбуку Морзе (к примеру)? Тоже ждем 20 минут и принимаем "онлайн".
    2. Экстраполируйте Вашу схему в ситуацию передачи данных с условной звезды, которая находится на расстоянии хотя бы 100 тыс. световых лет до нас и с нее кто-то прямо сейчас передает данные. Ждем 100 тыс. лет и принимаем "онлайн"? Так ведь получается.


    1. Gibboustooth
      20.02.2018 01:19

      Автор думает, что системе есть какая-то разница, прочитаем ли мы это сообщение сами или оно будет записано на жесткий диск компьютера и мы его прочитаем через десять минут. На самом деле никакой разницы быть не может — если система работает так, как думает автор, сообщение можно читать прямо сразу, еще до того как фотоны прилетели в точку назначения и там начали делать какие-то манипуляции с зеркалом. И тут в полный рост вылезают все парадоксы, связанные с нарушением принципа причинности.


      1. bull1251 Автор
        20.02.2018 01:43

        если система работает так, как думает автор, сообщение можно читать прямо сразу, еще до того как фотоны прилетели в точку назначения и там начали делать какие-то манипуляции с зеркалом. И тут в полный рост вылезают все парадоксы, связанные с нарушением принципа причинности.

        В общем то можно решить эту проблему. Достаточно расположить даун-конверторы по середине, чтобы расстояние до экрана соответствовал расстоянию до ловушек. Тогда не придется хранить запись, а это решает парадоксы с нарушением принципа причинности. В случае с Марсом это будет конечно проблемно, но тоже вполне решаемо.


        1. Gibboustooth
          20.02.2018 02:25

          Любая передача информации быстрее скорости света нарушает принцип причинности. Просто в таком случае парадоксы не столь очевидны.


        1. BigBeaver
          20.02.2018 09:39

          Просто подумайте: если свет находится «везде» достаточно одновременно чтобы вот так рулить картинкой, то почему надо вообще ждать, пока долетят фотоны? Смешивать выводы из разных систем отсчета — опасная практика.


  1. coal
    20.02.2018 01:34

    К статье с сумасшедшинкой захотелось оставить такой же сумасшедший комментарий насчет волнового дуализма и возможного его объяснения.

    Все вокруг нас заполнено полями и мы в этих полях плаваем. Полей, как известно четыре, возьмем электромагнетизм для простоты (фотоны). Если не ошибаюсь, принято считать, что фотон ведет себя то как волна, то как частица — но что, если фотон всегда ведет себя как волна и он — это всегда волна. А вот фотон как частица — это некоторый артефакт «конденсации» волны при внесении в нее возмущения любого рода. Где возмущением является любая другая частица материи, которая конденсирует поле, заставляя его «выпадать в осадок», «уплотняться», «кристаллизоваться».


    1. alexzzam
      20.02.2018 12:01

      Если не ошибаюсь, принято считать, что фотон ведет себя то как волна, то как частица

      Ошибаетесь. Просто у нас есть интуитивные представления о «частицах» как маленьких упругих шариках и о волнах «типа как на воде». Реальная частица это сложный объект, который всегда ведёт себя присущим ему образом, причём часть его проявлений похожа на то как работает «частица-шарик», а часть на то как работает макро-волна «типа как на воде».
      Годный (кажется) способ представить это — «частица это самое маленькое возможное возмущение поля».


      1. coal
        20.02.2018 12:29

        Квант — квантом, однако эти самые маленькие возмущения четырех полей имеют свойство слипаться во вполне конкретный материальный мир. Который с точки зрения физики представляет собой всего лишь интерференцию полей. А вот почему поля слипаются в частицы — сие никому неведомо, именно с позиции «почему» (конечно я знаю о глюонах, но это механизм, а не объяснение).
        А представление «поле конденсируется» вполне доступно образно — где вся материя это выпавший осадок полей ))) Ну, в виде шутки, в которой возможно есть доля правды.


        1. BigBeaver
          20.02.2018 12:32

          Это все просто плохие метафоры.


          1. coal
            20.02.2018 12:49

            какие бы ни были образные метафоры — объяснения физика не дает на вопрос «почему это всё так». Описать можно терминологически верно, а вот можно ли понять почему физика сама по себе является парадоксальной и нарушает сама себя.

            Если не очень понятно где нарушает, то это законы термодинамики, которые говорят нам о том, что замкнутая система всегда стремится прийти в состояние равномерного равновесия (энтропия, тепловой кисель, абсолютный белый шум). То есть вся вселенная должна распадаться, с уменьшением структурности и информационной сложности, а не самоусложняться постоянно. Должна прийти в тепловой кисель, а не самоорганизовываться. Мы же наблюдаем обратное.
            Или вселенная не замкнутая система, но экспериментально это не обнаружено.

            Это я пишу к тому, что должна быть образующая сила для энергии (полей), которая заставляет ее «конденсироваться», «комковаться», «усложняться» и самоорганизовываться, нарушая свои собственные законы. На роль такой образующей силы как раз подходит информация — а вот здесь мы упираемся в то, что человечество только только начало работать с ней, в том числе в физических моделях.
            Если информация образующая структурирующая сила — то это не то, что передается, то есть не объект воздействия, а субъект, организующий энергию. Везде рассматривается передача информации посредством энергии, но мало кто пытался взглянуть с обратной стороны — на структурирование потока энергии через создание информации…
            В общем, человечество с информацией только начало работать и мало ли что там таится неизвестного, возможно и сверхсветовые скорости, как знать… И тогда автор статьи будет на коне, а комментаторы будут похожи на ученых 19 века, которые считали, что клетка внутри заполнена мистическим желе…

            И, скажите, зачем вы ставите минусы — от неумения выйти из шаблонного мышления и попытаться взглянуть под другим углом вне догматической веры?


            1. BigBeaver
              20.02.2018 13:31

              объяснения физика не дает на вопрос «почему это всё так».
              Она и не должна.
              нарушает сама себя.
              Где?
              Должна прийти в тепловой кисель, а не самоорганизовываться. Мы же наблюдаем обратное.
              Это лишь ошибка вашей интерпретации. Тепловая смерть вселенной (просто например) как раз и будет апогеем «теплового киселя». А до тех пор ничто не запрещает существовать локальным условно упорядоченным областям.
              которая заставляет ее «конденсироваться»
              а нет ни какой конденсации, это тоже ошибка интерпретации. Волна и частица не сменяют друг друга, а существуют одновременно (строго говоря, существуют не они, а некоторая надсущность). Просто в разных случаях лучше подходят разные формулы, вот и все.
              И, скажите, зачем вы ставите минусы
              Вам? Почему вы решили, что это я?
              от неумения выйти из шаблонного мышления и попытаться взглянуть под другим углом вне догматической веры?
              А вот за подобные оскорбления можно и по карме получить, но мне лень.


              1. coal
                20.02.2018 13:43

                А до тех пор ничто не запрещает существовать локальным условно упорядоченным областям.


                да, конечно, не запрещает. Весь вопрос почему на пути к тепловому киселю (вектор понятен, самораспад, например машина на стоянке через 1000 лет превратится в труху) — почему на этом пути происходит невероятно сложная самоорганизация материи… От химических элементов до мозга с самосознанием, которым мы все это пишем. Да что там мозг, простая клетка сложнее всего, что придумал человек…

                Физика, возможно, и не должна отвечать на этот вопрос, почему всё происходит вопреки ей — это скорее область философии. Однако, нельзя не признать, что многое еще непонятно и что ввод информации как полярной силы для энергии и организующей, структурирующей ее — это выглядит достаточно логично.

                Но вот в уравнениях Эйнштейна нет ничего про информацию, к сожалению… Да и вообще мало где есть, разве то в уравнениях у Хокинга про черные дыры.

                А может быть, всего лишь дописав в уравнения значок информации — получится получить единую теорию, что никак не выходит пока… И, может быть, этот значок и даст возможность сверхсветовых скоростей. И почему бы информации не являться причиной «комкования» полей со «слипанием» (за образную терминологию извиняюсь, но не в ней суть все же). Другими словами, мысль в том, что уравнения Эйншейна верны, но не являются полными.

                Вам? Почему вы решили, что это я?


                Тогда извините


                1. BigBeaver
                  20.02.2018 14:01

                  почему на этом пути происходит невероятно сложная самоорганизация материи
                  Просто флуктуации. Нужно понимать разницу между случайным распределением и равномерным. Равномерное распределение есть полная упорядоченность. То есть вселенная фотонов была более упорядочена чем наша существующая, хоть локально вы и видите примеры сложной организации.
                  Но вот в уравнениях Эйнштейна нет ничего про информацию, к сожалению
                  Это и не требуется. Информация не мыслима без чего-то материального. А на него выводы Эйнштейна очень даже действуют.
                  всего лишь дописав в уравнения значок информации
                  Для начала попробуйте определиться с терминологией)
                  P.S.
                  Тогда извините
                  Я вообще очень редко минусую комментарии, на которые отвечаю. Либо минус либо ответ.


                  1. coal
                    20.02.2018 14:06

                    Это и не требуется. Информация не мыслима без чего-то материального. А на него выводы Эйнштейна очень даже действуют.


                    Справедливо и обратное утверждение: материя (она же энергия по Эйнштейну) немыслима без чего-то информационного (описание состояния).

                    Вот этой информации и нет в уравнениях, как значка. Может потому и не удается объединенную теорию построить пока.

                    Для начала попробуйте определиться с терминологией)


                    Пускай будет волновой коллапс для моих терминов типа «конденсация». Суть то не особо меняется, как ни назови.


                    1. BigBeaver
                      20.02.2018 14:13

                      Я про термин «информация». Вот этот ваш комментарий содержит ее вообще ноль — ничего не дополняет и не опровергает.


                      1. coal
                        20.02.2018 14:24

                        почему же. Комментарий вполне информационный — он содержит мнение о том, что уравнения Эйнштейна неполны, так как описывают только поведение материи, но не включают в себя информацию. Почему ее надо включать — логический вывод предоставлен чуть ранее. Без этого компонента физика является парадоксальной.

                        Вы попытались выше устранить парадокс вот так:

                        Просто флуктуации. Нужно понимать разницу между случайным распределением и равномерным. Равномерное распределение есть полная упорядоченность. То есть вселенная фотонов была более упорядочена чем наша существующая, хоть локально вы и видите примеры сложной организации.


                        но давайте рассмотрим две крайних позиции: начало вселенной и конец вселенной.
                        Началом вселенной являлся полный абсолютный хаос без отсутствия каких-либо структур, я не ошибаюсь? Это и есть идеальная энтропия.
                        А вот с ходом времени этот абсолютный хаос все время непрерывно самоусложняется, причем до такой степени, что мы (структурированный хаос) задаемся вопросом «почему». Вы назвали это флуктуацией… Но посчитав вероятность этой флуктуации только математически на протяжении 15 млрд лет — мы получим возможно самое малое число из которых оперирует человек. Но пускай так, самое невероятное событие когда-то может случиться, пускай флуктуация.
                        Продолжаем мысленный эксперимент — флуктуация все усложняется и усложняется, в конце вселенной получаем тот же кисель, что вначале, только максимально однородный и с высокоорганизованными гениальными «флуктуациями» в ней, сложно даже представить какими (если погаснет звезда, это «флуктуация» зажжет ее заново). Ведь флуктуация не остановится, верно? С чего бы вдруг…

                        Итого: вначале имеем идеальный хаос и никакого порядка. В конце имеем сверхпорядок в виде «флуктуаций» посреди киселя.
                        Это и есть самоорганизация. И это абсолютно противоречит логике машины, оставленной на стоянке. Физике в принципе не известно самоорганизации систем. У вас ведь на столе приборы сами из радиодеталей не собираются…

                        Если вам известен конкретный эксперимент с самоорганизацией вещества, с усложнением структуры — напишите. А коли таких нет — то введение организующей силы «информация» вполне оправдано для устранения парадокса. Следовательно, верен и логический вывод про неполноту уравнений. Следовательно, возможность для превышения скорости света не закрыта.


                        1. BigBeaver
                          20.02.2018 14:28

                          До тех пор, пока материя является носителем информации, описание материи описывает информацию. Информация отделенная от материи не может взаимодействовать с материей и, соответственно, влиять на материальный мир => бесполезна => ваши рассуждения бессмысленны.

                          p.s. дальше первого абзаца читать не стал по, надеюсь, очевидным причинам.


                          1. coal
                            20.02.2018 14:36

                            в уравнениях Хокинга, описывающих черные дыры — есть математическое понятие «информация». Непонятно, почему почему вы отказываете в возможности дополнения других уравнений этим же понятием…

                            В любом случае, благодарю за беседу.


                            1. BigBeaver
                              20.02.2018 14:45

                              Я вам не зря предложил определиться с терминологией. Начните отсюда.


                              1. coal
                                20.02.2018 15:05

                                да, это примерно то, о чем я хотел сказать. Как и то, что это очень молодая ветвь физики, с 2003 года всего лишь. До тех пор рассматривалась физика энергии и передача информации посредством энергии — то, что обсуждается в статье и комментариях.

                                Но ровно на эту же физику можно взглянуть под другим углом на 180 градусов — как на создании потоков энергии при создании разных потенциалов информации (информационной напряженности). Это та же самая физика и ничего материального не отрицает. Но вот на стыке этих двух описаний одного и того же под разным углом могут получиться какие-то новые открытия и уравнения, как знать. В этом и заключается наука — в поиске нового, не отрицая старого. Сам-то я не физик, как вы уже поняли, но это простая логика.


                                1. BigBeaver
                                  20.02.2018 15:21

                                  Так это просто способы описания. Новых сущностей там не вносится, и материя с информацией все так же неразделимы.


                                  1. coal
                                    20.02.2018 15:33

                                    и, однако, энергетическое описание от Эйнштейна имеется, а информационного нет пока. Какой-то физик когда-то напишет эти уравнения и будет увековечен в учебниках. Может даже будете вы ))

                                    Но для этого надо все перевернуть вверх ногами, взглянуть совсем с другой стороны. Как в формулах Эйнштейна нет «символа информации», так в новых формулах должен отсутствовать «символ энергии». И это будет описание тех же самых процессов, но иначе.

                                    А потом объединить эти полярные уравнения воедино. Вам мое описание возможно кажется дилетантским, но ведь это просто иной взгляд, а не опровергание основ.


                                    1. BigBeaver
                                      20.02.2018 15:48

                                      Потому, что это бессмысленно.


                                    1. Shkaff
                                      20.02.2018 17:22

                                      Не очень понятно, как вы перепишете уравнения Эйнштейна с точки зрения информации, если информация — величина статистическая и квантовая, а уравнения Эйнштейна — детерминистические и классические.


                                      1. coal
                                        20.02.2018 17:35
                                        -1

                                        Да, это не очень понятно… Это сможет сделать только гений, который выйдет за рамки догм начала 20 века… К сожалению, догматичность веры одинакова — что в физике, что в религии… Умение же мыслить креативно настолько редко, что за него ставят минусы — когда человек размышляет, а не подражает.


                                        1. BigBeaver
                                          20.02.2018 17:51

                                          Минусы ставят за чушь всякую. Дело в том, что новое описание хоть и может давать новые следствия, старым данным они противоречить все ранво не могут. Обычно же люди об этом забывают и пускаютс во все тяжкие в своих фантазиях. А потом обижаются, когда им предлагают на землю вернуться, обвиняют всех окружающих в косности и тд.

                                          догматичность веры одинакова — что в физике, что в религии
                                          А вот от подобного впредь воздержитесь — это вкорне не корректное высказывание, недопустимое в приличном обществе. Физиком вы можете не быть, но основы научной методологии знать должны, а подобные слова это отрицают, выводя вас в подкласс простых глупых смертных, с которыми и разговаривать-то нет смысла.


                                          1. coal
                                            20.02.2018 18:06

                                            Основы научной методологии я знаю, но знаете ли ее вы…

                                            Теорема Байеса говорит нам, что наши убеждения достоверны настолько, насколько верны свидетельства в их пользу. Пользуясь этой теоремой я привел неоспоримые непосредственно наблюдаемые свидетельства на тему самоорганизации/энтропии, которые противоречат современной картине мира. На это вы ничего не смогли ответить внятно и аргументированно, просто игнорируя факт полного отсутствия самоорганизации с самоусложнением в физике.

                                            Другими словами, я аргументированно показал неполноту текущих научных знаний и предложил один из возможных способов устранить эту неполноту, выдвинув гипотезу, опять же по Байесу. Выдвижение гипотез является научным, а оснований для этого достаточно, написал их в первом абзаце, а также можно вспомнить про отсутствие «теории всего», что говорит опять же о неполноте текущих научных знаний.

                                            Защищая эту самую неполноту и оставаясь в ее рамках — вы никогда не выйдете за ее рамки. Именно это и есть догматичность и «подражание»

                                            Что касается опровержения чего-либо, то вы очень невнимательно читаете что я пишу. Прямым текстом написал, что «В этом и заключается наука — в поиске нового, не отрицая старого» и «рассмотрение проблемы под другим углом дает новый взгляд на то же самое», но вам было угодно подменить это на свое понимание. Вам просто так захотелось — это и есть ненаучно.


                                            1. BigBeaver
                                              20.02.2018 18:12

                                              Я вам ответил про самоорганизацию, и еще один товарищ тут тоже ответил. Вы же эти ответы проигнорировали.

                                              я привел неоспоримые непосредственно наблюдаемые свидетельства о самоорганизации
                                              просто игнорируя полное отсутствие наблюдений самоорганизации с усложнением
                                              Эмм…
                                              я аргументированно показал неполноту текущих научных знаний
                                              Как будто бы кто-то ее отрицал.
                                              но вам было угодно подменить это на свое понимание.
                                              Нет. Но да, я не особо внимателен к вашим комментариям. Это не очень хорошо, но мне почему-то лень вникать в кучу текста, которую я считаю слабо осмысленной.


                                              1. coal
                                                20.02.2018 18:20

                                                ну вы же просто троллите в виде «не хочу ничего читать, пишу что хочу».

                                                Аргумент про самоорганизацию простой — она наблюдается фактически 15 млрд лет. Но противоречит законам термодинамики и не проявляется ни в одном физическом опыте. Она не может быть и невозможна по любым текущим известным уравнениям. Она ими не описана и просто игнорируется.

                                                Это и говорит о неполноте знаний. Выйти из неполноты можно только строя гипотезы и проверяя их — это и есть наука. Гипотеза не должна опровергать существующее знание, а должна расширять — такая и была предложена.

                                                Догматичностью же является нежелание выходить из неполноты знаний и не признавать эту неполноту… И, конечно, подавлять «инакомыслие». И ни в коем случае не пытаться рассмотреть инородные вариации.

                                                Все просто.


                                                1. mihaild
                                                  20.02.2018 18:30

                                                  А с чего вы взяли, что энтропия Вселенной 10 миллиардов лет назад выше текущей? Скажем образование звезды и планетной системы из газопылевого облака вы считаете «самоорганизацией»? (этот процесс полностью согласуется с известными законами)
                                                  Если нет — то приведите конкретный пример «самоорганизации».


                                                  1. coal
                                                    20.02.2018 18:40

                                                    В данном случае вы правы, на орбите даже воспроизвели такую самоорганизацию в плазменном поле, частицы пыли стремились к центру масс, создавая «планету».

                                                    Но это и противоречит термодинамике — равномерному тепловому киселю. Он получается недостижим, хотя согласно ей обязан быть в конце всего.

                                                    Если же рассмотреть самоорганизацию химических веществ в информационные структуры вроде клеточных фабрик или нервной системы — это еще менее понятно, причем всем. То, что эволюция есть — это бесспорно. Но почему происходит самоусложнение систем, когда должен идти самораспад — вот это и есть предмет поиска, а для него надо переосмысливать существующие парадигмы, раз они не объясняют или противоречат.

                                                    Мы зашли уже очень далеко от тематики статьи. Я не утверждаю, что законы ОТО неверны, просто говорю о неполноте и о том, что рано постулировать невозможность превышения скорости света именно из-за этой неполноты.

                                                    Опять же, посредством гипотетических тахионов ее вполне можно превысить и это вполне научные гипотезы о происходящем за горизонтом событий черной дыры.


                                                    1. BigBeaver
                                                      20.02.2018 18:45

                                                      О боги, возьмите да посчитайте энтропию систем «вселенная со звездами» и «вселенная-кисель». Если вы не физик, то и не надо спорить с физиками. Вам указывают на ошибку в интерпретации, а вы в ответ обвиняете в троллинге, ну как так-то?

                                                      рано постулировать невозможность превышения скорости света
                                                      Ее никто и не постулирует. Она является следствием экспериментальных данных. Чтобы это узнать, достаточно даже коментов к этой публикации — не обязательно быть физиком.


                                                      1. coal
                                                        20.02.2018 18:51

                                                        О боги, возьмите да посчитайте энтропию систем «вселенная со звездами» и «вселенная-кисель».


                                                        Извините, это сделать невозможно, ввиду мистической «темной материи» — если не ошибаюсь, 80% от вселенной, и не менее мистической «темной энергии». Которые просто математическая абстракция, константа, введенная потому что нам «непонятно» и, конечно, не обнаруженные, а существующие только в фантазии, чтобы подогнать.

                                                        Но вы можете продемонстрировать что вы физик и доказать мне обратное расчетами.


                                                        1. mihaild
                                                          20.02.2018 18:56

                                                          Ну хорошо, посчитайте энтропию газопылевого облака, из которого образуется звездная система, и образовавшейся системы. На таких масштабах темной материей можно пренебречь.

                                                          Возникновение необходимости в «подгонке» — это и есть «обнаружение».


                                                          1. coal
                                                            20.02.2018 19:04

                                                            давайте еще упростим задачу: возьмем просто облако пыли на земле, посчитаем его энтропию. И сожмем облако в твердое тело и посчитаем энтропию. Такая аналогия сопоставима?

                                                            Если да, можно пойти дальше. Взять идеальный газ и посчитать его энтропию. Затем охладить до твердого состояния, собрать в кучку и посчитать энтропию.

                                                            Я не знаю что получится, но на газе считать проще. Это расчеты того же уровня?


                                                            1. BigBeaver
                                                              20.02.2018 19:12

                                                              Взять идеальный газ и посчитать его энтропию. Затем охладить до твердого состояния, собрать в кучку и посчитать энтропию.
                                                              Если не забыть включить в систему охлаждалку, то всё будет ок. А если забыть, то система не замкнута. и в ней вообще что угодно может быть.


                                                              1. coal
                                                                20.02.2018 19:26

                                                                Отлично. А кто охлаждает Вселенную? Она ведь закрытая система? Ах да, она же расширяется и остывает… В «осадок» из охлаждающегося газа выпадают звезды, планеты, амебы, люди… Прикольная конденсация или, как вы назвали, флуктуация, жаль с идеальным газом такое не провернуть. Не возникает в нем почему-то таких флуктуаций…


                                                                1. BigBeaver
                                                                  20.02.2018 19:28

                                                                  Вы бы так резко аналогиями не жонглировали, а то мало ли.


                                                                  1. coal
                                                                    20.02.2018 19:42

                                                                    Вы привели очень сильный аргумент, я сдаюсь и признаю свое поражение.


                                                                1. mihaild
                                                                  20.02.2018 19:35

                                                                  Когда газ коллапсирует под действием гравитации, он нагревается, его энтропия уменьшается, но энергия тоже уменьшается. Куда девается лишняя энергия?



                                                        1. BigBeaver
                                                          20.02.2018 19:05

                                                          Если это невозможно, то на основании чего вы заявляете, что нарушены законы термодинамики?


                                                          1. coal
                                                            20.02.2018 19:16

                                                            я рассматриваю не энергетическую энтропию, а информационную энтропию, как меру неопределенности. Есть такое понятие, как раз по моей профессии… И вот она уменьшается 15 млрд лет.

                                                            Но это не аргумент, а просто риторическое замечание. Ведь все, что связано с информацией физика просто игнорирует — это пока за ее пределами.

                                                            Энергетическую энтропию посчитать нельзя, растет ли, уменьшается ли… Но опираясь на информационную энтропию, аналогию вполне можно провести. Даже просто логически — раз сложность систем растет, значит энтропия уменьшается.


                                                            1. Shkaff
                                                              20.02.2018 19:21
                                                              +1

                                                              я рассматриваю не энергетическую энтропию, а информационную энтропию, как меру неопределенности.

                                                              Наверное я вас расстрою, но нет такой энтропии как "энергетическая". Вся энтропия как в информатике, так и в физике определяется одинаково, через распределения вероятностей (хоть по Шеннону, хоть по Нейманну).


                                                              Ведь все, что связано с информацией физика просто игнорирует — это пока за ее пределами.

                                                              Вы просто не знаете, как обстоят дела в физике.


                                                              1. coal
                                                                20.02.2018 19:32
                                                                -1

                                                                Вся энтропия как в информатике, так и в физике определяется одинаково, через распределения вероятностей

                                                                Странно, а здесь написана формула без всяких вероятностей…

                                                                ispu.ru/files/u2/book2/TD1_19-06/ttd4-1-4.htm

                                                                Вы точно физик? Три раза уже вы написали противоположное общепринятому знанию и упрекаете при этом меня в незнании физики…


                                                                1. Shkaff
                                                                  20.02.2018 19:38

                                                                  Вы бы еще по школьному учебнику смотрели определения физических величин. Эта формула выводится из статистической физики, где определяется через распределения вероятностей, ровно как в информатике.


                                                                  Я точно физик, у меня и справка есть, и гугл знает.


                                                            1. mihaild
                                                              20.02.2018 19:23
                                                              +2

                                                              Если при проведении аналогии получается противоречие с хорошо известными законами физики — скорее всего проблема в аналогии, а не в физике.


                                                            1. BigBeaver
                                                              20.02.2018 19:26

                                                              По-моему, вы сейчас произвели очень резкую подмену темы. Сначала вы сказали, что поведение энтропии во вселенной противоречит законам физики. Потом вы сказали, что вообще-то речь о другой энтропии, которую физика не изучает.

                                                              Но подумайте сами, если физика не затрагивает ее, то и законы физики она нарушать не может. Значит, в ваших рассуждениях противоречия даже, если я соглашусь, что ваши утверждения об информационной энтропии вселенной верны (но разумеется, я не соглашусь, и жду от вас доказательства).


                                                    1. mihaild
                                                      20.02.2018 18:49
                                                      +2

                                                      Да не противоречит. Энтропия планетной системы со звездой выше, чем энтропия газопылевого облака, из которого она образовалась.

                                                      Если же рассмотреть самоорганизацию химических веществ в информационные структуры вроде клеточных фабрик или нервной системы — это еще менее понятно, причем всем.
                                                      Ну да, для этого понадобился бы какой-то внешний (по отношению к Земле) источник энергии, причем мощный. Скорее всего термоядерная реакция на протяжении миллиардов лет. А вдруг такой есть?


                                        1. Shkaff
                                          20.02.2018 18:22

                                          Послушайте, дело в догмах, а в том, что это просто о разном. ОТО не имеет ничего общего с информацией, не описывает ее, и не будет описывать ее. Проблема в совмещении квантов и ото, но это фундаментальная проблема.


                                          Ваша "креативность" — на уровне старшей школы, а пафос — как будто вы нобелевский лауреат, отсюда и минусы.


                                          1. coal
                                            20.02.2018 18:46

                                            В вашем блоге приведена вот такая картинка. Прямо с волнами

                                            habrastorage.org/files/e7d/480/b84/e7d480b848684aa08e602151ea58cddf.jpg

                                            Получается, вы мой сторонник, ведь с этого всё началось, что «волны — не волны, а просто абстракция».


                                            1. Shkaff
                                              20.02.2018 19:01

                                              Не понял, как это относится к вашим рассуждениям. Я пишу про гравитационные волны, и картинка про них. Гравитационные волны не квантуются в ОТО.


                                              1. coal
                                                20.02.2018 19:10

                                                странно, а вот тут «школьник» прочитал про гипотетический квант гравитационного поля — гравитон
                                                ru.wikipedia.org/wiki/%D0%93%D1%80%D0%B0%D0%B2%D0%B8%D1%82%D0%BE%D0%BD

                                                Более того, этот гравитон входит в стандартную модель физики элементарных частиц из 61 частиц, как совершенно официальная точка зрения современной науки.

                                                Да и если подумать — почему три поля квантуются и имеют переносчики, а гравитационное нет. С чего бы…

                                                А то, что в ОТО про это не сказано — эта теория неверна что ли… Зачем на нее ссылаться в данном случае… Скорее всего волновую природу имеют все поля одинаковую (гипотеза) и отделять ваши красивые картинки от ОТО вряд ли правомерно.


                                                1. BigBeaver
                                                  20.02.2018 19:14

                                                  а вот тут «школьник» прочитал
                                                  Казалось бы, при чем тут ОТО…
                                                  А то, что в ОТО про это не сказано — эта теория неверна что ли…
                                                  Нет, это просто разные теории, описывающие разные вещи.


                                                1. Shkaff
                                                  20.02.2018 19:15
                                                  +1

                                                  Как все запущенно… Вы каким местом читаете?


                                                  Гравитация не квантована, гравитон — гипотетическая частица, которую никто не знает, как привязать к любой из теорий. Она не часть стандартной теории точно, не знаю, откуда вы это взяли. Цитирую ссылку, которую вы мне дали: "Попытки расширить Стандартную модель гравитонами сталкиваются с серьёзными теоретическими сложностями в области высоких энергий (равных или превышающих планковскую энергию) из-за расходимостей квантовых эффектов (гравитация не ренормализуется)."


                                                  Да и если подумать — почему три поля квантуются и имеют переносчики, а гравитационное нет. С чего бы…

                                                  Над этим бьются уже почти сто лет, но никто не знает. Не существует пока теории, которая бы квантовала гравитацию, не содержала ошибок и при этом была применима к нашему миру.


                                                  1. coal
                                                    20.02.2018 19:21
                                                    -2

                                                    Она не часть стандартной теории точно, не знаю, откуда вы это взяли.

                                                    Об этом «школьник» прочитал здесь
                                                    ru.wikipedia.org/wiki/%D0%A1%D0%BF%D0%B8%D1%81%D0%BE%D0%BA_%D1%87%D0%B0%D1%81%D1%82%D0%B8%D1%86

                                                    там внизу и источники есть. Гравитон гипотетическая частица, согласен.

                                                    Над этим бьются уже почти сто лет, но никто не знает. Не существует пока теории, которая бы квантовала гравитацию, не содержала ошибок и при этом была применима к нашему миру.

                                                    Да, вот это я тоже хочу сказать и в этом вы тоже мой соратник. Нам пока неизвестно многое и нестыковок много. Но без гипотез — они решены не будут. Специальные дядьки за деньги в теоретической физике изобретают всякую чепуху — почему с ними не спорите-то…


                                                    1. Shkaff
                                                      20.02.2018 19:25

                                                      Гравитон гипотетическая частица, согласен.

                                                      Ну а чего тогда городить? Известны только его гипотетические свойства, но никто не знает, как его получить.


                                                      Но без гипотез — они решены не будут.

                                                      Только обоснованных и математически проверенных гипотез, а ваша "гипотеза" — это уровень старшей школы в лучшем случае, как мы уже выяснили.


                                                      Специальные дядьки за деньги в теоретической физике изобретают всякую чепуху — почему с ними не спорите-то…

                                                      Вы такой умный, сразу видите, что это чепуху они изобретают, а мы тут все сидим и работаем над этим каждый день, и вот как то никак.


                                                      1. coal
                                                        20.02.2018 19:37

                                                        Ну а чего тогда городить? Известны только его гипотетические свойства, но никто не знает, как его получить.

                                                        вы же заявили, что гравитационное поле не квантуется. Но теперь поправились, что просто пока экспериментально квант не обнаружили (слишком высокие энергии нужны).

                                                        Так уже намного лучше. А то, знаете, голословные утверждения о несуществовании чего-то делать для ученого хуже смерти.


                                                        1. BigBeaver
                                                          20.02.2018 19:39

                                                          Когда говорят «но никто не знает, как его получить.» речь о принципиальных способах. То есть, нет способа построить научный эксперимент для его обнаружения. Значит, утверждения о его существовании не научны.


                                                          1. Shkaff
                                                            20.02.2018 19:41

                                                            Я имел ввиду получить в теоретическом описании. Как в "получить решение уравнения".


                                                        1. Shkaff
                                                          20.02.2018 19:40
                                                          +1

                                                          Нет, вы не понимаете. Гравитационное поле не получается проквантовать, тк любое решение расходится в области высоких энергий.
                                                          Никто никогда не получал описание гравитона, которое бы работало и с гравитацией, и с квантовой теорией.


                                                          1. Zenitchik
                                                            20.02.2018 22:57
                                                            +1

                                                            «Попытки расширить Стандартную модель гравитонами сталкиваются с серьёзными теоретическими сложностями в области высоких энергий (равных или превышающих планковскую энергию) из-за расходимостей квантовых эффектов (гравитация не ренормализуется).»


                                                            Но теперь поправились, что просто пока экспериментально квант не обнаружили (слишком высокие энергии нужны).


                                                            Как тут не вспомнить анекдот про «У тебя негр родился, Геной назвали»…

                                                            Очевидно, coal не владеет терминологией, поэтому понял цитату… Очень странным образом.


                                                    1. BigBeaver
                                                      20.02.2018 19:32

                                                      Об этом «школьник» прочитал здесь
                                                      Там такого нет. Там написано «Кроме того, в других моделях вводятся следующие пока не зарегистрированные частицы:».


                1. dTex
                  20.02.2018 14:32

                  есть Синергетика, там рассматривается, как происходит самоорганизация материи. Насколько помню суть в том, что это и есть результат рассеивания энергии, просто структурированная материя рассеивает энергию лучше. Даже что-то про эволюцию кристаллов было, в которой побеждает и растёт тот, кто способен пропустить через себя больше энергии.


      1. Gibboustooth
        20.02.2018 14:08

        Есть нравящаяся многим физикам интерпретация квантовой механики в которой таки есть частицы «как маленькие упругие шарики» и волны «типа как на воде». Называется теория волны-пилота (Pilot Wave Theory). Вот интересное видео на эту тему.


        1. BigBeaver
          20.02.2018 14:13

          Так это лишь модель. К «как на самом деле» это не имеет отношения. Впрочем, оно всех моделей касается.


        1. bull1251 Автор
          20.02.2018 15:41

          Теория пилотной волны не объясняет, почему исчезает интерференция, когда наблюдают за частицей. Даже узнав, через какую щель прошел фотон, согласно теории пилотной волны мы должны получить интерференцию на экране:
          image


          1. Shkaff
            20.02.2018 17:27

            Нет, в теории пилотной волны факт измерения изменяет волновую функцию, так что при наблюдении за частицей интерференция пропадает. Так как эта теория — всего лишь интерпретация КМ, она не вносит новых эффектов в классическую КМ и наблюдаемое нами, просто объясняет их другим способом.


            1. Gibboustooth
              20.02.2018 21:10

              Делает ли теория пилотной волны предсказания, которые мы можем проверить?


              1. Shkaff
                20.02.2018 21:15

                Новых, отличных от стандартной квантовой механики — нет, она просто описывает те же явления другим языком. Самое полезное в ней — она объясняет "коллапс" волновой функции без всяких парадоксов. Но это просто то, как мы объясняем наблюдаемое, ни математика, ни собственно физика не меняются.


  1. Zmiy666
    20.02.2018 01:37

    Вообще очень странно Возможно возможностей моего мозга просто не хватает, чтоб осознать сие но как можно посмотреть в будущее или нарушить причинно-следственные связи??
    Вот вы чиркнули спичку — она загорелась.
    Вы мгновенно прыгнули на одну световую минуту и можете снова наблюдать как чиркаете спичку и она загорается… и снова прыгаете на минуту и еще раз наблюдаете эту картину…
    Но если вы прыгнете к началу то найдете там только сгоревшую спичку. Все события происходят одномоментно для всей вселенной и если событие уже произошло, его не отменить.
    Скорость света просто указывает максимальную скорость распространения конкретного носителя информации о событии в условиях этой вселенной. Информация расходится как круги на воде от источника события, но никак не дает возможности повлиять на событие. Но нет никаких логических причин, по которым нельзя было бы передвигаться и передавать информацию быстрее.
    мы ведь видим свет звезд, которые давно мертвы, если мы перенесемся туда, к звезде мгновенно — то можем не найти там ничего, включая саму звезду, которая за это время уже успела свалить в другую галактику. Но никаких законов мы не нарушим, парадоксов не создадим и тд


    1. mihaild
      20.02.2018 01:55

      Передача информации быстрее скорости света = передача информации в прошлое.
      Стандартный пример с поездом: поезд проходит мимо станции, причем с точки зрения станции передний вагон оказывается у переднего края станции в тот же момент, что задний у заднего, и по ранее достигнутой договоренности в этот момент сидящие по краям платформы товарищи стреляют в первый и последний вагоны. События происходят одновременно, отреагировать на одно до того как произойдет второе нельзя.
      С точки зрения поезда же в первый вагон стреляют раньше, чем в последний. Если внутри поезда есть сверхсветовая связь, то после выстрела в первый вагон можно передать информацию в последний до того, как в него попадут, и там успеют подготовиться.
      В итоге с точки зрения платформы использовать информацию об одном выстреле для подготовки к другому невозможно, а с точки зрения поезда — возможно. Проблема.

      Все события происходят одномоментно для всей вселенной
      Собственно вот в этом ошибка. Одновременность относительна — нельзя универсальным образом сказать, какие событие одновременны, а какие нет.


      1. Zmiy666
        20.02.2018 04:35

        эм…
        все равно не понял…
        в первый вагон выстрелили… потом в первом вагоне увидели вспышку выстрела в конце и отправили мгновенное сообщение о том, что в него стреляют? Ну так и придет оно туда даже не моментально, а моментально + время необходимое чтоб свет дошел от последнего вагона к первому… каким образом оно должно попасть в прошлое?
        Даже если они отправят сообщение мгновенно, не дожидаясь второго выстрела сзади, они ничего не смогут изменить, ведь выстрел то уже был — просто свет до них еще не дошел…
        Это лишь иллюзия возможности таймпарадокса.


        1. Gibboustooth
          20.02.2018 05:04

          Если поезд двигается относительно станции с релятивистскими скоростями, то для наблюдателей на станции он кажется короче. Мы можем так подобрать длину и скорость поезда, что для наблюдателей на станции его длина в момент прохождения мимо них совпадет с длиной станции и события «передний конец поезда пересёк передний край станции» (1) произойдет одновременно с событием «задний конец поезда пересек задний край станции» (2).

          Для наблюдателя в поезде, наобот, сокращаться в размерах будет станция. То есть, поезд окажется существенно длиннее станции и событие (1) наступит раньше события (2) (тут никакого парадокса нет, т.к. одновременность относительна).


          1. Zmiy666
            20.02.2018 08:20

            хм… как-то это тяжело воспринимается… жаль нету мультика который бы показал сей процесс наглядно.
            И что будет если наблюдатель в последнем вагоне будет знать о точном времени на станции, когда поезд займет точное положение. Он сверит часы (ну или получит сверхсветовое сообщение о стрельбе из первого вагона) и откроет оба окна в тот момент, когда по времени станции первый вагон займет позицию для стрельбы.
            Что будет с пулей и окном? Ведь оба выстрела были произведены одновременно, но по времени поезда была разница в N долей, в этот промежуток окно было открыто. Но для наблюдателей на станции оно не было открыто, ведь стреляли одновременно или нет? Пуля пробьет стекло или пролетит сквозь открытые окна? Или что вообще произойдет в адский промежуток?


            1. ARechitsky
              20.02.2018 08:55

              Одновременность относительна. См. парадокс шеста и сарая
              Если я не ошибаюсь, ОТО определяет одновременность так: отправим из точки А в точку Б фотон, а в точке Б он мгновенно отразится обратно. Середина временного интервала между отправкой и получением фотона в А одновременна с отражением в точке Б. Если точки А и Б движутся друг относительно друга — это искажает одновременность.


              1. Zmiy666
                20.02.2018 09:50

                хех, спасибо, стало более понятно :))
                Однако выходит, что все эти свойства характерны только при взаимодействии тел имеющих скорость, близкую к скорости света. И только если объект физический… принципиального противодействия перемещению информации и точки, а в току б мгновенно нет… только принципиальный запрет на подобное перемещение физического объекта-носителя информации… фотона там или иной частицы…


                1. BigBeaver
                  20.02.2018 09:59

                  А как вы себе представляете информацию, полностью отделенную от материального мира?

                  Ок, допустим, она существует, и ее можно передать (даже не важно, с какой скоростью). Но чтобы вы могли увидеть мою информацию, нужно сделать ее материальной, тк человек (и компьютер) воспринимает своей сенсорикой только материальное. Это значит, что моя нематериальная информация должна как-то изменять параметры (импульс, энергия, заряд и тд) материальных вещей на вашей стороне. Звучит, как бред, не находите?


                1. vedenin1980
                  20.02.2018 10:17

                  принципиального противодействия перемещению информации и точки, а в току б мгновенно нет

                  Есть, время на корабле замедлится относительно Земли, время на Земле замедлится относительно корабля, при мгновенной передачи информации начинаются парадоксы.

                  Представьте себе видеоконференцию когда на корабле видят, что каждая минута на Земле соответствует неделе на корабле, а на Земле наоборот. Парадокс.

                  Для мгновенной передачи придется отказаться от относительности времени и СТО/ОТО, где это главный постулат. А теории которая бы могла полностью заменить ОТО и позволяла мгновенную передачу пока не видно


                1. Gibboustooth
                  20.02.2018 14:19

                  Однако выходит, что все эти свойства характерны только при взаимодействии тел имеющих скорость, близкую к скорости света.


                  Эти свойства характеры для любых тел — законы физики не зависят от скорости. Просто на малых скоростях релятивистские эффекты принебрежимо малы.


                1. mihaild
                  20.02.2018 14:38

                  Что такое «не физический объект» — не знаю. В разных системах отсчета время идет по-разному, и одновременные в одной СО события могут быть не одновременны в другой.

                  Да, все эти эффекты пропорциональны гамма-фактору, который при скоростях, малых по сравнению со скоростью света, очень мал.
                  Неплохой «мультик» есть — www.youtube.com/watch?v=bWBr2E3Y3aY
                  Но если интересно, то лучше не ограничиваться такими изложениями «на пальцах», а разобраться с геометрией Минковского и преобразованиями Лоренца (там на самом деле не требуется ничего, кроме математики школьного уровня и отказа от интуиции). Есть хорошая книжка «Физика пространства-времени» Тейлора, Уилера и чуть более сложный (но и более краткий) 2й том Фейнмановских лекций по физике.


                  1. Zmiy666
                    20.02.2018 17:10

                    хм… спс за инфу, пожалуй самое главное это — отказ от интуиции — а то она постоянно говорит, что так не бывает, а если применить математику, то внезапно все становится логично…


        1. mihaild
          20.02.2018 14:32

          Нет, в первом вагоне увидели, что стреляют в первый вагон, и отправили сообщение в последний. С точки зрения поезда в первый вагон выстрелили раньше, чем в последний — и если разрешить передачу информации со сверхсветовой скорости, то в последнем вагоне узнают о том, что в первый стреляли, до того, как выстрелят в последний вагон.


          1. Gibboustooth
            20.02.2018 14:35

            И таким образом получится, что для наблюдателя на странции наблюдатель в последнем вагоне поезда узнал будущее.


      1. pr0gz
        20.02.2018 12:13

        Если с Земли на корабль летящий со скоростью близкой к скорости света, то в прошлое (т.к время на корабле замедляется относительно земли). Если наоборот, то получается что в будущее. При этом будет интересный эффект. Допустим имеется мгновенная связь на любом расстоянии и договорились что с Земли постоянно будут отсылать на корабль какую-то информацию. Он разгоняется до скорости при которой действует большое замедление времени. Плотность потока данных (байт/c) при этом станет такой огромной, что компьютеры на корабле скорее всего не смогут ее обработать. А вот плотность данных с корабля на землю наоборот сильно снизится. Т.е использование такой связи во время полета на скоростях близких к скорости света будет мягко говоря затруднительно. По крайней мере мне так кажется)


  1. vanxant
    20.02.2018 06:47

    Автор, в статью не вникал, но осуждаю пару советов «на будущее».
    1. Не надо лохматить дедушку Эйнштейна) Его многие пытались поправить, но все обломали зубы. Трюк в том, что Эйнштейн говорил про передачу действия («запрет ужасающего дальнодействия»), т.е. сверхсветовую передачу энергии и импульса. Но вам это не нужно, ваши фотоны несут свою энергию с положенной им от рождения скоростью света.

    1б. Но лохматить Эйнштейна разговорами про «телевизор, показывающий будущее» таки не надо.

    2. Речь может идти только о мгновенной передаче квантового состояния запутанных частиц. Это давний наброс на вентилятор, еще со времён ЭПР-парадокса и неравенств Белла. Однако, сегодня экспериментально доказано, что это работает (в смысле, передача состояния происходит мгновенно).

    3. Тем не менее, большинством физиков считается, что передавать таким образом информацию нельзя. Так же, как нельзя передавать информацию при помощи солнечного зайчика, хотя технически он может двигаться быстрее света. Но, вообще говоря, это ни откуда не следует, потому что принцип причинности, если его читать буквально, то есть по буквам в-л-и-я-н-и-е (т.е. «воздействие, оказываемое на какой-либо объект и приводящее к изменению его свойств, положения и т. п.»), не нарушается. Есть, например, «обратимые (слабые) квантовые измерения», и если они действительно работают, то мгновенная передача таки возможна. Вопрос в том, как придумать проверяемую схему эксперимента.

    4. На возражения Tyusha про деление на ноль можно предложить замену фотонов на электроны со скоростью 0,99999c. В вашем эксперименте скорость частиц несущественна, правда, придётся заменить оборудование на спинтронное.
    Впрочем, если решите этим заняться, нужно очень аккуратно считать скорости. Можно случайно взять расстояния из одной системы отсчёта (например, вашего электрона) и время из другой (в системе внешнего наблюдателя). Числа тут могут получиться очень привлекательные, только физического смысла в них не будет.


    1. Shkaff
      20.02.2018 13:20

      Есть, например, «обратимые (слабые) квантовые измерения», и если они действительно работают, то мгновенная передача таки возможна. Вопрос в том, как придумать проверяемую схему эксперимента.

      Нет, таки невозможна. Слабые измерения — очень хитрый класс измерений, применим к малому количеству систем, и чудес он не позволяет.


  1. fedorro
    20.02.2018 12:54

    Осталось атомный\субатомный ассемблер-дизассемблер придумать, и можно будет производство телепортов налаживать. Офисы на Марсе, курорты в районе Альфа Центавра — красота =)


  1. rvt
    20.02.2018 13:53

    Вспомнил так называемые «кажимости». Я еще когда учился в универе на астронома, мы «проходили» наблюдения сверхсветовых скоростей. Как нам объяснили (ошибка в определении скоростей исключалась), это типа светового зайчика, который движется по экрану.
    Теоретически такое световое пятно при достаточно большой угловой скорости источника и достаточно большом «плече» (расстоянии от источника до экрана) может двигаться со сверхсветовой скоростью.


  1. madf
    20.02.2018 17:23

    Болтология какая-то, чтобы увеличить скорость передачи информации, достаточно параллельно проложить N количество оптических кабелей (пример надеюсь понятен). :D Но тут возникает сразу другая проблема, чем вы собираетесь обрабатывать эти данные, передать и принять — не проблема. А вот в зависимости от принятых данных совершать сразу действия и отправлять обратно данные (реалтайм) — средств пока таких известных массовому потребителю — нет.